2Окт

Дросселирующее устройство это: Дросселирующее устройство (регулятор давления после себя, регулятор давления до себя, регулятор расхода)

Содержание

Дросселирующее устройство — Большая Энциклопедия Нефти и Газа, статья, страница 1

Cтраница 1

Дросселирующие устройства представляют собой различные гидравлические сопротивления, служащие для уменьшения расхода или давления в какой-то системе или в определенных ее частях. Например, проходя через не полностью открытую задвижку или другое подобное препятствие, поток теряет часть своей энергии. На рис. 19, г показана картина огибания потоком выступающей задвижки. Перед задвижкой наблюдается типичное сужение потока, за задвижкой — расширение. Потери давления вычисляют по формуле ( 48), причем коэффициент местного сопротивления t, зависит от степени открытия задвижки, меняясь от незначительной величины при полностью открытой задвижке до бесконечности при закрытой задвижке.  [1]

Схема редукционно-охладительной установки ( РОУ-I, РОУ-2 и.  [2]

Дросселирующие устройства Dy20 — 50 мм могут устанавливаться как на горизонтальных и вертикальных участках трубопроводов подвода охлаждающей воды к охладителям пара РОУ и БРОУ, так и на линиях циркуляции обратных вертикальных клапанов, устанавливаемых на питательных насосах.

 [3]

Дросселирующее устройство одновременно является эффективным пылеуловителем и действует аналогично трубе Вен-турй. Скорость газового потока в дросселирующих устройствах достигает 250 — 320 м / сек. Их устанавливают как перед электрофильтрами, так и после них. В последнем случае электрофильтры рассчитывают для работы под повышенным давлением и дросселирование чистого газа предусмотрено в этих схемах временно — до установки газовых турбин, в которых в дальнейшем может быть использована энергия сжатого газа.  [4]

Дросселирующее устройство перед флотатором выполняется в виде диафрагмы с расширяющимися по ходу движения воды конусом.  [5]

Винтовой расходомер. 1 — корпус. 2 — струе. аы.  [6]

Дросселирующие устройства исполняются трех видов ( рис. 6): острая диафрагма ( наиболее распространенная), сопло и труба Вентури. При прохождении среды через суженное отверстие увеличивается скорость потока, часть потенциальной энергии потока переходит в кинетическую. Величина перепада давления ( Рг и Р2) до и после сужения зависит от количества протекающего газа или жидкости, что дает возможность вычислить их расход.  [7]

Основные технические данные и размеры дросселирующих устройств БРОУ 43 ЭМ.| Огнозные технические данные и размеры охладителей пара БРОУ ЧЗЭМ.  [8]

Дросселирующие устройства различных исполнений отличаются расчетными параметрами пара, полной длиной L, размером LI и массой.  [9]

Все дросселирующие устройства должны быть снабжены указателями степени их открытия и иметь фиксаторы для закрепления в различных положениях.  [10]

Все дросселирующие устройства должны снабжаться указателями степени их открытия или закрытия и иметь фиксаторы для закрепления устройств в различных положениях. Управление высоко расположенными регулирующими устройствами ведется на высоте не более 1 7 м от пола или специальной площадки. Необходимо систематически следить за исправностью всех вентиляционных решеток и в особенности регулируемых, где повреждения поворотных устройств или отдельных перьев может вызвать уменьшение объема подаваемого или удаляемого воздуха.  [11]

Все дросселирующие устройства должны быть снабжены указателями степени их открытия или закрытия и иметь фиксаторы для закрепления устройств в различных положениях. Управление высоко расположенными регулирующими устройствами ведут на высоте не более 1 7 м от пола или специальной площадки. Необходимо систематически следить за исправностью всех вентиляционных решеток и в особенности регулируемых, где повреждения поворотных устройств или отдельных перьев может вызвать уменьшение объема подаваемого или удаляемого воздуха.  [12]

Какие дросселирующие устройства устанавливаются на линиях непрерывной продувки испарителей и парообразователей.  [13]

Управляющие дросселирующие устройства интересующих нас типов состоят из дросселей переменного и постоянного сечений, которые соединяются таким образом, что могут в соответствии с требованиями изменять сопротивление потоку жидкости, подаваемой от источника питания к гидродвигателю при перемещении управляющего элемента в зависимости от какого-либо внешнего сигнала.

Будем считать, что о характере нагрузки нам ничего не известно и что величина перепада давлений на гидродвигателе рт и расход через него qm могут независимо принимать любые значения вплоть до максимального. Нашей задачей является составление эквивалентной схемы для каждого типа дросселирующего устройства и его рабочего режима, а также вывод на основе этой схемы функциональной зависимости между рт, qm, положением штока х ( или другого входного сигнала) и известными постоянными величинами.  [14]

Наличие дросселирующего устройства в гидроцилиндре позволяет регулировать скорость опускания наклонной части стояка. В верхней части гидроцилиндра 2 расположена гидрозащелка, предназначенная для фиксирования наклонной части наливного стояка в крайнем верхнем положении и препятствующая ее самопроизвольному опусканию. Изменение расстояния между герметизирующей крышкой и осью вертикальной стойки стояка в пределах от 2 25 до 3 05 м обеспечивается лопастным гидроприводом 6 и механизмом изменения вылета стояка.

 [15]

Страницы:      1    2    3    4

Дросселирующие устройства в холодильных установках

Процесс дросселирования, то есть понижение давления, является неотъемлемым элементом холодильного цикла. Именно благодаря этому процессу и существует разница давления между зоной высокого давления и зоной низкого давления холодильного контура. Элемент холодильной системы, в котором происходит процесс дросселировния, называется дросселирующим устройством или расширительным устройством.

 

Рис. 1. Процесс дросселирования на диаграмме.

Дросселирование хладагента обеспечивается многократным снижением пропускной способности дросселирующего устройства относительно канала подачи хладагента (жидкостной линии). Это снижение может быть как постоянным, так и изменяемым (регулируемым). Существует несколько типов расширительных устройств.

Виды расширительных устройств.

Нерегулируемые

Как следует из названия, нерегулируемые дросселирующие устройства создают постоянное сопротивление движению хладагента и не реагируют на изменение режимов работы холодильной машины. Нерегулируемое дросселирующие устройство подбирается заранее и, как правило, обеспечивает эффективную работу холодильной системы, только в каком-то одном режиме.

Конструктивно существует большое количество различных нерегулируемых расширительных устройств: дросселирующие шайбы, жиклеры, дюзы и т.д. Однако, самым распространенным, в практических холодильных установках, является дросселирующее устройство в виде капиллярной трубки.

Рис.2. Капиллярная трубка в составе холодильного контура.

                Капиллярная трубка создает сопротивление движению хладагента не столько за счет меньшего диаметра канала, сколько за счет значительного удлинения. Как правило, степень дросселирования капиллярной трубки точно регулируется именно длиной трубки, а не изменением её сечения. В некоторых системах длина капиллярной трубки может достигать нескольких метров, что позволяет отмерять её длину при помощи обычной рулетки или линейки с достаточным уровнем точности.

Рис. 3. Медная капиллярная трубка.

                Основным недостатком капиллярной трубки, как и любого нерегулируемого дросселирующего устройства, является неспособность адаптироваться к изменению режима работы холодильной машины. Любое дросселирующее устройство постоянного действия поддерживает только лишь перепад между зонами высокого и низкого давления, но не конкретное значение.        

                В случае, если необходимо поддерживать стабильное давление кипения хладагента, давление конденсации на агрегатах с капиллярными трубками так же должно быть стабильно. Снижение давления конденсации ниже расчетного повлечет за собой снижение давления кипения и общее снижение производительности агрегата, тогда как повышение давления повлечет снижение перегрева и риск гидроудара компрессора.

                Так же постоянные дросселирующие устройства не могут контролировать значение перегрева хладагента на линии всасывания компрессора. Эффективная и безопасная работа установки может обеспечиваться только в  небольшом диапазоне внешних условий.

                Как следствие, нерегулируемые расширительные устройства применяются только в составе небольших агрегатов, работающих в постоянных режимах: бытовых холодильниках, простых кондиционерах и т.д.

Терморегулирующие расширительные вентили (ТРВ)

Для решения описанных недостатков нерегулируемых расширительных устройств, разработаны различные модели дросселирующих устройств регулируемых. Наибольшее распространение среди механических устройств получит так называемый Терморегулирующий расширительный вентиль или ТРВ.

Рис.4. ТРВ в составе холодильной системы.

                Конструктивно ТРВ состоит из дюзы (канала определенного сечения, через который проходит хладагент), запорной иглы (устройства, перекрывающего движение хладагента через дюзу) и термостатирующей системы.

Термостатирующая система, в свою очередь, состоит из мембраны, на которую с одной стороны воздействуют давление хладагента в зоне низкого давления и специальная пружина, а с другой стороны мембрана соединена с термобаллоном (небольшая металлическая капсула, внутри которой находится хладагент, аналогичный заправленному в систему).

 Рис.5 Конструкция ТРВ

                Термобаллон ТРВ устанавливается на линии выхода хладагента из испарителя, где воспринимает на себя температуру выходящего хладагента. В том случае, если давление в термобаллоне (а значит и температура газа на выходе из испарителя) превосходит давление хладагента на значение, определяемое силой пружины, ТРВ открывается и подает хладагент в испаритель. В том случае, если температура газа на выходе снижается, ТРВ ограничивает подачу хладагента в испаритель.

                С точки зрения физических параметров, ТРВ регулирует перегрев хладагента на выходе из испарителя независимо от остальных параметров работы установки.

Электронные дросселирующие устройства

Отдельным пунктом стоит выделить расширительные устройства, управляемые электронными системами: Электронные расширительные вентили или ЭРВ.

ЭРВ контролируют перегрев газа на выходе из испарителя аналогично ТРВ. Различие заключается в методе измерения параметров установки. Если ТРВ использует исключительно механический способ регулирования подачи хладагента, то в состав ЭРВ входи специальный микроконтроллер, воспринимающий информацию о температуре газа и давлении от специальных датчиков. Датчики ЭРВ устанавливаются на контур аналогично термобаллону ТРВ и линии выравнивания.

Контроль работы дросселирующих устройств.

Любое дросселирующие устройство имеет своей целью ограничивать поток хладагента в испаритель таким образом, что бы, с одной стороны, не допустить попадания жидкого хладагента в компрессор, а, с другой стороны, обеспечить максимальное заполнение испарителя.

Основным параметром, определяющим корректность работы дросселирующего устройства, является перегрев хладагента на выходе из испарителя. В системах с нерегулируемым дросселирующим устройством повлиять на перегрев возможно только косвенно – изменяя давление конденсации или количества хладагента в системе.

Таким образом, для контроля работы ТРВ необходимо измерить перегрев.

Высокое значение перегрева может иметь различные причины, поэтому, прежде чем регулировать ТРВ, необходимо убедиться, что изменение перегрева не вызвано недостатком хладагента или потерями давления на жидкостной линии.

Низкое значение перегрева всегда свидетельствует о некорректной работе ТРВ.

Регулировка ТРВ и ЭРВ.

Регулировка ТРВ

В том случае, если значение перегрева отклоняется от номинального, а все иные возможные причины исключены, производится регулировка ТРВ. Настройка ТРВ осуществляется поворотом регулировочного винта.  

Рис.6. Регулировочный винт ТРВ.

                В зависимости от применяемого хладагента и модели вентиля, поворот винта на один оборот может привести к различным изменениям в работе ТРВ. В том случае, если реакция вентиля на вращение регулировочного винта неизвестна, не рекомендуется поворачивать регулировочный винт более чем на один оборот за один прием.

 Рис.7. Вращение регулировочного винта по часовой стрелке увеличивает перегрев. Вращение против часовой стрелки – уменьшает.

                После настройки ТРВ регулировочным винтом, новые замеры перегрева целесообразно производить не ранее чем через 20 минут. В противном случае, перегрев может не успеть стабилизироваться.

Регулировка ЭРВ

ЭРВ, как цифровое устройство, не требует как такового регулирования. Будучи единожды настроенным, микроконтроллер будет поддерживать заданное значение без отклонений.

Ошибки в работе ЭРВ могут быть вызваны либо некорректной первичной настройкой, либо выходом из строя одного из элементов.

Процесс контроля работы ЭРВ сводится к сравнению показаний датчиков давления и температуры с эталонными.

Приглашаем Вас на обучение по курсам: 

ХП1 – Ремонт и обслуживание холодильного оборудования

На курсе вы обучитесь ремонтировать и производить диагностику холодильников, морозильных камер, ларей, а так же полупромышленных холодильных установок. По окончанию обучения Вы получите удостоверение установленного образца.

ХП3 – Ремонт и сервисное обслуживание холодильного оборудования

Данный курс в первую очередь будет полезен для сотрудников сервисных служб и рабочего персонала связанного с холодильным оборудованием

ПХ2 — Сервис и техническое обслуживание холодильного оборудования, работающего на природных хладагентах

Курс предназначен для специалистов с опытом ремонта бытового и полупромышленного холодильного оборудования. По итогу обучения вы получите удостоверение установленного образца, который дает разрешение на обслуживание данных холодильных установок.

В теоретической и практической части обучения, мы расскажем вам о новейших технологиях ремонта и монтажа оборудования. А так же во время практических работ мы предлагаем современные инструменты и новые методики работы.

Для изучения теории слушателям, мы предложим учебное пособие: Экологические аспекты, безопасная эксплуатация, сервис и обслуживание холодильного оборудования и систем кондиционирования воздуха от 2020 года. Пособие подготовлено специалистами нашего учебного центра.

Подробнее о датах практических занятий Вы можете узнать в  разделе Расписание.

Дросселирующие узлы

Дросселирующие устройства в однолинейном представлении являются узлами, но во внутренней кодировке — это дополнительные участки с постоянным или переменным сопротивлением. В дросселирующий узел обязательно должен входить только один участок, и только один участок из узла должен выходить.

Внимание: Исключение из данного правила составляют регуляторы давления которые используют вспомогательный участок. В этом случае из регулятора давления выходит два участка — один основной и один вспомогательный.

Дроссельная шайба

С точки зрения модели дроссельная шайба это фиксированное сопротивление, определяемое диаметром шайбы, которое можно устанавливать как на подающем так и на обратном трубопроводе.

Так как это нерегулируемое сопротивление, то величина гасимого шайбой напора зависит от квадрата проходящего через шайбу расхода. На рисунке видно, как меняются потери на шайбе, установленной на подающем трубопроводе, при увеличении расхода через нее в два раза.

Регулятор давления

Регулятор давления — это устройство с переменным сопротивлением, которое позволяет поддерживать заданное давление в трубопроводе в определенном диапазоне изменения расхода. Регулятор давления может устанавливаться как на подающем так и на обратном трубопроводе.

На рисунке показано, что при увеличение в два раза расхода через регулятор, установленный в обратном трубопроводе,  давление в регулируемом узле остается постоянным. Величина сопротивления регулятора может изменяться в пределах от бесконечности до сопротивления полностью открытого регулятора. Если условия работы сети заставляют регулятор полностью открыться, то он начинает работать как нерегулируемый дросселирующий узел.

Регулятор располагаемого напора

Работа регулятора  располагаемого напора аналогична работе регулятора давления только в этом случае регулятор старается держать постоянной заданную величину располагаемого напора.

Регулятор расхода

Регулятор расхода — это узел с переменным сопротивлением, которое позволяет поддерживать постоянным заданное значение проходящего через регулятор расхода. Регулятор можно устанавливать как на подающем так и на обратном трубопроводе. К работе регулятора расхода можно отнести все сказанное про регуляторы давления.

Вспомогательный участок

Режим 1 — вспомогательный участок для регулятора давления

По умолчанию Регулятор давления регулирует давление в том месте, где установлен. Вспомогательный участок предназначен для того, чтобы узел контроля за регулируемым параметром для регулирующего устройства мог быть задан самим пользователем. На рисунке ниже показан участок трубопровода на котором установлен регулятор давления регулирующий давление после насосной станции, но контролирующий давление перед насосной станцией.

При указании узла контроля необходимо учитывать, что он обязательно должен быть

простым узлом.

Режим 2 — вспомогательный участок для ЦТП

В случае, если после ЦТП вода на систему отопления и вода на ГВС выходит по разным трубопроводам можно воспользоваться вспомогательным участком. Данный вспомогательный участок работает только со схемой ЦТП №17. Он предназначен для того, чтобы указать трубопровод подающий теплоноситель на систему отопления и трубопровод подающий воду на систему горячего водоснабжения.

Следует ли при вводе сети точно повторять конфигурацию участков на местности? С точки зрения информативности и наглядности это желательно. Однако для гидравлических расчетов важна не конфигурация участка трубопровода, а следующие два условия:

  • Каждый участок должен соединять нужные узлы, т.е. сеть нужно описать топологически корректно.

  • Каждому участку должны быть заданы табличные параметры, позволяющие правильно определить его гидравлическое сопротивление.

Н а рисунке изображены два способа задания одного и того же участка тепловой сети. Верхний участок соединяет две камеры прямой линией. Нижний участок соединяет эти же две камеры, но линия выполнена с прорисовкой П-образных компенсаторов. Геометрическая длина двух участков различна и наличие компенсаторов влияет на сопротивление участка. Но в расчетах длина не вычисляется по рисунку, а задается в таблице по участкам. Особенности конфигурации участка (компенсаторы, углы поворота, ответвления и т.д.) учитываются в таблице в виде суммы коэффициентов местных сопротивлений. Поэтому, если мы для обоих участков, несмотря на их внешние различия, зададим в таблице одинаковую длину и одинаковый суммарный коэффициент местных сопротивлений, то сопротивление этих участков в расчетах будет одинаковым. Заметим только, что для равенства сопротивлений, у участков должны быть равны и другие табличные параметры: диаметры, шероховатости и зарастания.

Этот пример показывает, что точность отображения сети на карте на результаты расчетов не влияет.

Можно наносить сеть на точную карту города с соблюдением масштаба, можно в качестве подложки использовать немасштабный план местности, а можно вводить сеть схематично вообще без подложки.

Устройство кондиционера

В этой статье мы попробуем в доступной форме описать устройство кондиционера. Из школьного курса физики, мы помним, что при испарении, влага забирает тепло, а при конденсации, отдает. Простейший пример, стоит пролить на руку немного спирта или одеколона, как руке сразу станет холодно. Это происходит из-за быстрого испарения жидкости. Обратный процесс почувствовать сложнее, потому что влага, например над кипящей кастрюлей, не только конденсируется на руке, но и тут же испаряется.

Именно на этом принципе и базируется устройство кондиционера. Во внутреннем блоке происходит кипение и испарение хладагента (фреон – газ, кипящий при комнатной температуре и атмосферном давлении). Фреон забирает тепло у теплообменника внутреннего блока, который еще называется испаритель. Воздух, прогоняемый вентилятором через испаритель, отдает свое тепло и выходит из блока охлажденным. Куда же девается забранное у воздуха тепло?

Во внешнем блоке, который находится на улице, происходит обратный процесс. Под давлением, создаваемым компрессором, хладагент конденсируется в теплообменнике внешнего блока, который называется конденсатор. Размеры и характеристики испарителя и конденсатора подбираются таким образом, чтобы весь фреон в них успевал полностью превратиться в жидкость или газ. Вообще все устройство кондиционера рассчитано под конкретную мощность и в сплит-системах разной мощности практически не бывает одинаковых деталей.

Компрессор представляет собой насос высокого давления для газа. Компрессор создает как раз такое давление, чтобы при нормальных температурах весь хладагент успевал сконденсироваться во внешнем блоке. Далее хладагент проходит через дросселирующее устройство. В бытовых кондиционерах это капиллярная трубка. Именно такую трубку можно увидеть под теплообменником на задней стенке старого холодильника. Кстати холодильник устроен точно так же, как и кондиционер.

В капиллярной трубке давление падает и хладагент начинает кипеть. Но так как все трубы холодильного контура надежно утеплены, существенной потери производительности кондиционера не происходит. Основной теплообмен совершается при попадании кипящего хладагента в испаритель, обдуваемый теплым воздухом. Интенсивность кипения повышается лавинообразно и так же быстро понижается температура теплообменника.

Поэтому устройство кондиционера представляет собой следующее. Внешний блок – это металлический ящик с вентилятором и соответствующими отверстиями, в котором находится компрессор, капиллярная трубка, вентилятор внешнего блока, а также конденсатор. Еще во внешнем блоке «теплого» кондиционера размещается четырехходовый клапан, который позволяет обернуть процесс вспять и заставить кондиционер обогревать воздух не снаружи, а внутри. Также во внешнем блоке инверторного кондиционера размещается плата управления компрессора. В последнее время получили распространение сплит-системы с генератором кислорода. В этом случае рядом с компрессором размещается мембрана и вакуумный насос кислородного генератора.

Внутренние блоки кондиционеров бывают в разных форм-факторах. Самые распространенные – настенные «мыльницы». Также бывают кассетные, канальные, напольные, подпотолочные, колонные, угловые и т.д. И как бы причудливо не выглядел внутренний блок сплит-системы, он содержит испаритель и вентилятор. Также во внутреннем блоке размещается соответствующая электроника.

Вентилятор применяется диагональный и представляет собой полый цилиндр, стенки которого составлены из крыльчатки. Воздух проходит сквозь вентилятор и через диффузор выбрасывается в комнату. Такие вентиляторы при небольших размерах и низком уровне шума позволяют продувать через себя довольно большое количество воздуха. Но есть один минус такого устройства: диагональные вентиляторы не способны преодолевать сколько-нибудь значительное сопротивление воздушному потоку. Поэтому если установить во внутреннем блоке плотный фильтр, задерживающий очень мелкие загрязнения, воздух просто не будет проходить сквозь него.

Испаритель как бы огибает вентилятор. Забор воздуха происходит как с лицевой части настенного блока, так и сверху. При охлаждении воздух происходит конденсация влаги из него. Если не отводить конденсат, из внутреннего блока польется вода. Для отвода конденсата под нижними частями испарителя установлены ванночки, из которых вода стекает в дренажную трубку. Чаще всего дренажная трубка выводится на улицу, куда конденсат сливается самотеком. Скорость конденсации воздуха зависит как от мощности кондиционера, так и от влажности в помещении.

Устройство кондиционера не заканчивается на внутреннем и внешнем блоке. Сплит-система не может работать без соединительных коммуникаций. Это две медные трубы для жидкости и газа в теплоизоляции, кабели связи и питания. Обычно все коммуникации при монтаже скрепляются монтажным скотчем в жгут, куда также входит и дренажная трубка. Такой жгут имеет диаметр 4-5 см и может быть уложен в пластиковый короб или замурован в стену.

Устройство кондиционера делает его сложным агрегатом, который весьма капризно ведет себя при некачественном монтаже. После соединения всех коммуникаций из них необходимо откачать весь воздух, для того, чтобы в контуре не осталось кислорода и влаги, которые заставляют детали компрессора ржаветь и медленно убивают его. Также при наличии влаги в контуре кондиционер иногда начинает обмораживать испаритель.

Поэтому грамотно установить кондиционер смогут только специалисты, обладающие необходимыми инструментами и навыками. А некачественный монтаж способен загубить даже самую дорогую и надежную технику. Гарантийные условия распространяются только на правильно смонтированную технику, поэтому доверять монтаж следует компаниям, которые не только помогут с выбором, но и доставят, установят и в случае поломки сами ее устранят.

Принцип работы кондиционера в квартире: описание, характеристики

Содержание

Дело в том, что холод не «производится», а происходит перенос тепла из одного места в другое с помощью хладагента. Благодаря этому и появился термин «тепловой насос». По этой же причине кондиционер «производит» тепла или холода примерно в 3 раза больше, чем потребляет электроэнергии — факт, вызывающий недоумение у людей, не обремененных знанием холодильной техники.

Понять, как устроен кондиционер и откуда в тридцатиградусное пекло берется освежающая прохлада, не так уж сложно. Рассмотрим это на примере сплит-системы. Как известно из школьного курса физики, при испарении любая жидкость поглощает тепло. Если налить на руку спирт или одеколон, тут же почувствуешь холод. И наоборот, при конденсации пара тепло выделяется. Именно этот известный принцип и эксплуатирует любой кондиционер.

Дело в том, что холод не «производится», а происходит перенос тепла из одного места в другое с помощью хладагента. Благодаря этому и появился термин «тепловой насос». По этой же причине кондиционер «производит» тепла или холода примерно в 3 раза больше, чем потребляет электроэнергии — факт, вызывающий недоумение у людей, не обремененных знанием холодильной техники.

Что за чудо — машина с КПД 300%? И почему это загадочное вещество «хладагент» то поглощает, то отдает тепло, ведь из школьного курса физики известно, что оно всегда переходит от более нагретого тела к менее нагретому? Что заставляет хладагент переносить тепло из помещения, в котором чуть больше 20 градусов на улицу, где порой бывает под +40?

Но для того, чтобы кондиционер заработал, в замкнутый контур нужно встроить еще как минимум два элемента. Это компрессор, повышающий давление до давления конденсации, который установлен в контуре перед конденсатором, и дросселирующее устройство, понижающее давление до давления испарения, перед испарителем.

Перечисленные пять элементов:
1. замкнутый контур с хладагентом,
2. наружный теплообменник,
3. внутренний теплообменник,
4. компрессор,
5. дросселирующее устройство,
составляют основу холодильного контура любого кондиционера, от самого простого до самого сложного.

Для того, чтобы кондиционер мог работать не только на холод, но и на тепло, в контур необходимо добавить четырехходовой вентиль. Его задача «превращать» испаритель в конденсатор и наоборот. Такой кондиционер называют кондиционером с реверсивным циклом, который может переносить тепло не только из помещения на улицу, но и наоборот.

Кондиционер и уличный воздух:

Принцип работы кондиционера в квартире

Суть данного процесса заключается в том, чтобы забрать комнатный воздух и «пропустить» через внутренний блок (а точнее через его радиатор, где воздух охладится). Затем охлажденный воздух возвращается обратно в помещение. Так происходит до тех пор, пока температура в комнате не достигнет нужного значения.

Осуществляется это следующим образом:

  1. У кондиционеров «on/off» (неинверторных).

Компрессор кондиционера работает на «полную» (воздух в комнате охлаждается), когда температура в помещении теплее, чем нужно. Как только температура достигает нужного значения, компрессор выключается. Таким образом, кондиционер то включается, то выключается, поддерживая настроенную температуру.

Компрессор такой сплит-системы работает на «полную» мощность, если температура в помещении намного теплее, чем нужно. Когда температура приближается к настроенному значению, компрессор «сбавляет обороты» и с минимальной мощностью поддерживает «градус».

Это нужно знать: вал (вентилятор) внутреннего блока вращается независимо от работы компрессора.

фильтры тонкой очистки;

Вы удивитесь, но кондиционер работает очень просто. Для сравнения: достаточно капнуть на кожу воду, пока она испаряется, вы ощущаете холод. Принцип устройства бытового кондиционера такой же: хладогент, циркулируя, охлаждается и нагревается. Разбираемся, как работает кондиционер в квартире, что такое сплит-система и из чего она состоит.

Кондиционер иначе называется сплит-системой (от англ. split — разделение). Он состоит из двух блоков: наружного и внутреннего. Первый крепится на стене со стороны улицы, второй — в помещении. Вместе они составляют замкнутую систему движения хладагента — рабочего вещества кондиционера.

Внешний и внутренний блок сплит-системы (кондиционера)

Основные узлы кондиционера

компрессор. Сжимает хладагент, который в процессе сжатия нагревается, и поддерживает его движение по холодильному контуру;

конденсатор. Радиатор, находящийся в наружном блоке. В нём газообразный фреон,охлаждаясь за счет обдува уличным воздухом, конденсируется до жидкого состояния;

дросселирующее устройство. Понижает давление, уменьшая температуру хладагента;

испаритель. Радиатор, находящийся во внутреннем блоке. Противоположен по принципу действия конденсатору: в нем фреон при повышении температуры испаряется;

вентиляторы. Обеспечивают теплообмен внутреннего и внешнего блоков кондиционера с окружающей средой;

медные трубки, по которым циркулирует хладагент.

Устройство наружного блока

Наружная часть кондиционера размещается на стене со стороны улицы. Это позволяет обеспечить отведение тепла из помещения и снизить уровень шума в нем. Наружный блок состоит из нескольких базовых частей:

вентилятор. В простых моделях имеет одну скорость вращения. Дорогие варианты предполагают несколько скоростей либо плавную регулировку;

конденсатор. В домашних кондиционерах используются конденсаторы воздушного охлаждения;

компрессор. Встречаются классические роторные и двухроторные. Вторые отличаются практически полным отсутствием вибраций.;

дросселирующее устройство. В домашних кондиционерах имеет вид капиллярной трубки или электронного расширительного вентиля;

плата управления. Во внешнем модуле встречается лишь на инверторных кондиционерах.

Устройство внутреннего блока

Внутренний блок — часть сплит-системы, которая располагается в комнате. В него входят:

декоративная передняя панель корпуса. За ней скрываются воздушные фильтры и испаритель (теплообменник).

фильтр грубой очистки. Сетка, которая ловит на входе во внутренний блок крупные частицы: шерсть, волосы, пыль;

фильтры тонкой очистки;

вентилятор. В отличие от вентилятора наружного блока, как правило, имеет несколько скоростей;

испаритель. Представляет собой медную трубку с алюминиевым оребрением;

горизонтальные жалюзи. Подвижные элементы блока, которыми можно управлять при помощи пульта. Они направляют поток воздуха в нужную сторону по вертикали;

вертикальные жалюзи. Почти во всех бытовых сплит-системах их можно регулировать лишь вручную. Направляют воздушный поток по горизонтали;

блок управления. В защищенном пластиковом боксе установлена плата управления, к которой подключены пусковые элементы двигателей и датчиков;

индикаторная панель. Располагается на передней панели кондиционера и демонстрирует состояние работы оборудования;

дренажная ванночка. Лоток, в который стекает конденсат с поверхности теплообменника. Затем конденсат отводится в канализацию или на улицу по дренажной трубке.

Внутренних блоков в квартире можно установить несколько. При этом они могут быть подключены к одному наружному блоку.

Принцип работы кондиционера

Основа процесса — особенность жидкостей поглощать тепло при испарении и отдавать его при конденсации. В кондиционере циркулирует хладагент, который в зависимости от температуры и давления меняет агрегатное состояние, то есть становится то газом, то жидкостью.

Четыре базовых узла – компрессор, конденсатор, дросселирующее устройство и испаритель – объединены между собой трубками, создающими холодильный контур, внутри которого движется хладагент.

Принцип работы кондиционера

Из испарителя в компрессор поступает газообразный хладагент с низкой температурой. В компрессоре производится сжатие газа, одновременно повышаются и давление, и температура газообразного хладагента. Далее горячий хладагент под большим давлением поступает в конденсатор.

В конденсаторе газ остывает, т.к. обдувается потоком более холодного воздуха и превращается в жидкость, отдавая тепло. А выходящий из конденсатора воздух нагревается за счет тепла, отданного сжатым хладагентом.

Далее хладагент поступает в дросселирующее устройство. На этом участке он несколько теплее атмосферного воздуха, находится в жидком состоянии и под высоким давлением. В процессе прохождения через дросселирующее устройство давление хладагента резко снижается. Это сопровождается понижением его температуры.

В завершении цикла хладагент возвращается в испаритель, где на него воздействует комнатный воздух. Хладагент вновь становится газом, забирая тепло из комнаты. И такой цикл повторяется до тех пор, пока кондиционер включен.

Как избежать поломок?

Пожалуй, одна из самых сложных проблем при работе сплит-системы – это не успевший перейти в газ хладагент. Из испарителя жидкость попадает на вход компрессора, а поскольку жидкость – несжимаемое вещество, происходит гидроудар. Основная причина поломки – в загрязнении фильтров внутреннего блока, засорении тополиным пухом поверхности конденсатора внешнего блока. Поэтому нужна профилактика: регулярная очистка и плановые осмотры специалистом.

Если соблюдать простые правила, сплит-система прослужит долгие годы, обеспечивая надежное и безупречное кондиционирование воздуха.

Для переноса тепловой энергии кондиционер потребляет электроэнергию. Но следует отметить, что кондиционер переносит приблизительно в три раза больше энергии, чем потребляет. Электроэнергия необходима для работы компрессора, который создавая перепады давлений заставляет хладагент то испарятся то конденсироваться.

Внутренний блок состоит из следующих основных узлов:

Следует отметить, что кондиционер не производит холод (или тепло), а переносит его из помещения на улицу. В зависимости от выбранного режима, кондиционер либо переносит тепло из помещения на улицу, либо с улицы в помещение (обогрев).

Чтобы охладить воздух в комнате, необходимо отвести тепло, полученное в результате охлаждения. Тепло — это энергия. А энергия, как известно, не может исчезнуть бесследно. Именно по этому кондиционер состоит из двух блоков: внутреннего и наружного. Существуют также одноблочные системы охлаждения, которые отводят тепло по выведенному наружу воздухопроводу.

Переносчиком тепловой энергии в кондиционере (как и в холодильнике) является специальный хладагент. Чаще всего им является фреон.

Параллельно парообразный хладагент проходит через компрессор наружного блока, он сжимается за счет работы компрессора, давление и температура (до 50-60°C) повышаются. Далее горячий пар, попадая в наружный блок, охлаждается и преобразуется в жидкость, отдавая свое тепло атмосфере. После конденсатора хладагент уже в жидком виде опять дросселируется, её давление падает, и температура вновь опускается до 5-10°C, жидкость снова начинает закипать в теплообменнике-испарителе, поглощая тепло из охлаждаемого помещения.

Практически вся современная техника работает от электричества (электросети или аккумулятора) – будь то холодильники, телевизоры, или встроенные пылесосы. Кондиционеры, хотя и не все из них являются бытовыми приборами, тоже работают от электрической сети. У них – непростая задача преобразовать получаемую рабочую энергию в прохладу и комфорт. И они, именно для этого их и проектировали, с этой задачей справляются.

Каков же, собственно, принцип работы кондиционеров?

Мы выяснили, что кондиционер потребляет электричество, но что же делает выходящий из него воздух таким холодным? И где этот воздух берется, кстати, тоже не совсем понятно.
Если спросить, откуда берется этот воздух у несведущего, мы в 9 из 10 ответов получим вариант, согласно которому воздух «всасывается из того большого шумного ящика с вентилятором снаружи» (внешнего блока кондиционера распространенного типа — бытовой настенной сплит-системы). Оставшийся ответ, скорее всего, будет просто «не знаю».

Но, давайте по порядку.

В случае с климатической техникой главную роль играет хладагент. Основываясь на его физико-химических свойствах, а точнее, изменение его давления для преобразования из жидкообразного состояния в пар и обратно, такие климатические устройства как кондиционер, способны разделять в пространстве области с пониженной и повышенной температурой. Такой же принцип работы заложен и в привычном нам холодильнике.

До относительно недавнего времени в кондиционерах, которые продаются на территории СНГ, в основном использовался хладагент R-22 (гидрофтороуглерод HCFC), теперь под влиянием природоохранных законов и общественного мнения производители постепенно переходят на типы хладагентов, не разрушающие озоновый слой Земли, такие как R-410 и R-407.

Параллельно парообразный хладагент проходит через компрессор наружного блока, он сжимается за счет работы компрессора, давление и температура (до 50-60°C) повышаются. Далее горячий пар, попадая в наружный блок, охлаждается и преобразуется в жидкость, отдавая свое тепло атмосфере. После конденсатора хладагент уже в жидком виде опять дросселируется, её давление падает, и температура вновь опускается до 5-10°C, жидкость снова начинает закипать в теплообменнике-испарителе, поглощая тепло из охлаждаемого помещения.

Являясь устройством, поддерживающим заданный микроклимат, кондиционер выполняет различные функции — охлаждения, очищения, нагрева воздуха, вплоть до осушения. Но в отличие от остальной техники, выполняющей свои функции при помощи воды, таких, например, как климатические комплексы, кондиционеры работают с влагой, получаемой непосредственно из воздуха.

Одна из основных и самых нагруженных деталей сплит-системы — это компрессор. Непосредственно участвует в процессе переноса тепла: приводит в движение фреон и осуществляет основную работу по изменению физических свойств хладагента. На схеме №2 изображён рабочий цикл кондиционера:

В основе работы любой системы кондиционирования, будь то домашняя сплит-система или промышленный кондиционер, заложены простые законы физики, которые гласят, что при испарении вещества происходит поглощение энергии и наоборот: при конденсации – выделение теплоты.

Любой современный кондиционер – это насос по перекачке тепла из одного места в другой путём чередования процесса расширения и сжатия рабочего тела (газа фреона).

На схеме №1 представлены основные агрегаты кондиционера:

2. Компрессор кондиционера.

Одна из основных и самых нагруженных деталей сплит-системы — это компрессор. Непосредственно участвует в процессе переноса тепла: приводит в движение фреон и осуществляет основную работу по изменению физических свойств хладагента. На схеме №2 изображён рабочий цикл кондиционера:

Как видно из представленной схемы работы кондиционера испарившийся (и, напомним, поглотивший в помещении тепло) фреон по фреонопроводу попадает в компрессор внешнего блока. Газообразный фреон имеет малое давление и достаточно высокую температуру, но для повышения КПД компрессор сжимает газ при этом температура ещё больше увеличивается на фоне роста давления, что увеличивает перепад температур воздуха снаружи теплообменника внешнего блока и рабоичм телом, т.е. самим фреоном.

В решётке внешнего блока сплит-системы фреон, имеющий избыточное давление и температуру быстро отдаёт тепло и переходит в жидкое состояние. Его температура понижается и он готов снова попасть во внутренний блок Вашей сплит-системы для удаления тепла.

Компрессор бывает разной конструкции: поршневой, ротационный, винтовой, спиральный. Классический поршневой компрессор из-за нестабильности создаваемого давления (цикличность работы приводит е перепадам давления рабочего газа) и шумности работы постепенно уходит в небытие.

Неоспоримые лидеры в производстве надёжных и долговечных компрессоров — производители исконно японских марок, таких как Mitsubishi Electric, Daikin, Toshiba.

Некоторые известные корейские производители кондиционеров, такие Panasonic, самостоятельно не производят компрессора, но закупают агрегаты у японских производителей и без страха дают гарантию в 5 лет на их работу. Новые современные компрессора, которые обладают увеличенным ресурсом, а также позволяют сплит-системе работать при существенных перепадах температур.

Компрессор зачастую работает на повышенных оборотах и под воздействием негативных внешних факторов. В состав фреона производители добавляют специальное масло и присадки для смазки особо нагруженных частей. Именно поэтому зачастую невозможно на старую фреоновую трассу (по которой проходил фреон типа R22) установить сплит-систему нового типа (на фреоне R410).

3. Теплообменник внешнего блока кондиционера.
Как уже понятно из названия (и описания предыдущих агрегатов) обеспечивает отвод тепла от газообразного фреона высокого давления для возможности перехода фреона в жидкое состояние.

Задача инженеров заключается в том, чтобы в ограниченном пространстве внешнего блока сплит-системы обеспечить отвод тепла, получаемого от целой комнаты/помещения в несколько квадратных метров. Это достигается благодаря извилистой форме трубок теплообменника, дополнительным вставкам пластин, принимающих на себя тепло.

Очень сильно на теплоотдачу влияет степень загрязнённости теплообменника. В случае если он засорится (из-за эксплуатации в пыльных условиях, налипания пуха и пыльцы), то резко снизится КПД теплоотдачи, что может привести к выходу всей системы из строя.

Теплообменник требует особого ухода и регулярной чистки. Более подробно Вы можете ознакомится в разделе «Обслуживание кондиционеров».

4. Фреонопровод.

Часть сплит-системы, которая собственно и позволила разделить в кондиционере функцию охлаждения (в помещении) и отвода тепла (на улицу). Состоит, как правило, из двух медных труб: одна меньшего диаметра — жидкостная, по ней фреон в жидком состоянии поступает во внутренний блок; вторая — большего диаметра, для отвода испарившегося и превратившегося в газ фреона из внутреннего блока кондиционера к теплообменнику наружного.

При прокладке трассы также укладывают:

Более подробно об особенностях и порядке монтажа можете узнать из раздела » Монтаж кондиционеров «.

5. Теплообменник внутреннего блока сплит-системы.

Призван обеспечить эффективное испарение фреона. При испарении фреона происходит охлаждение медной трубы теплообменника. Чем мощнее кондиционер — тем больше по габаритам приходится выполнять сам теплообменник. Именно поэтому с ростом мощности охлаждения сплит-системы увеличиваются и габариты самого внутреннего блока кондиционера.

  • осушение теплообменника. После выключения сплит-системы происходит временный обдув решётки для осушения
  • обработка теплообменника спец.покрытием, предотвращающим образование микроорганизмов

6. Крыльчатка внутреннего блока кондиционера.

Функция аналогична вентилятору внешнего блока. Особые требования относятся к аэродинамике и выравниванию воздушного потока внутреннего блока для достижения двух целей:

  • снижения шумового давления (для обеспечения комфортного пребывания в помещении при работающем блоке)
  • повышения коэффициента полезного действия при теплообмене
  • настенные блоки;
  • настенно-потолочные;
  • кассетные устройства;
  • канальные;
  • колонные кондиционеры;
  • оконная техника;
  • шкафные (прецизионные) устройства.

Существующие типы кондиционеров

Сегодня можно подобрать устройство, которое будет поддерживать нужную температуру в помещении наряду с учетом особенностей конфигурации помещения. При этом можно выбрать модель подходящей мощности и с соответствующим интерьеру внешним видом. Принцип работы кондиционера в ряде случаев одинаков, однако, некоторые типы предполагают индивидуальные конструктивные особенности. Различают:

  • настенные блоки;
  • настенно-потолочные;
  • кассетные устройства;
  • канальные;
  • колонные кондиционеры;
  • оконная техника;
  • шкафные (прецизионные) устройства.

В момент испарения происходит поглощение теплоты в помещении, которая через конденсатор выбрасывается наружу.

Устройство бытовой сплит-системы

Стандартная классическая сплит-система состоит из следующих частей:

Внутренний блок включает в себя следующие компоненты: корпус, испаритель – в нем происходит переход фреона из жидкого состояния в газообразное, воздушный центробежный вентилятор, предназначенный для интенсивного теплообмена с окружающей средой, воздушные фильтры (в зависимости от модели кондиционера), приспособления для специальной обработки воздуха, жалюзи и поддон для сбора конденсата (рисунок 2).

Переносчиком тепловой энергии в кондиционере (как и в холодильнике) является специальный хладагент. Чаще всего им является фреон.

Принцип работы кондиционера. Как работает кондиционер?

Начнем с простого. Вещества имеют такое свойство — при испарении они поглощают тепло, при конденсации они его выделывают. На этом физическом явлении и построена работа кондиционера.
Принцип действия кондиционера основан на изменении агрегатного состояния хладагента (фреона) в зависимости от температуры и давления в замкнутой системе. Для начала ознакомимся с основными узлами кондиционера.

Основными узлами любого кондиционера являются:

У современного кондиционера имеется электронный блок, который управляет работой в зависимости от выбранного режима. Все современные модели имеют пульт дистанционного управления, позволяющий задавать параметры.

Наружный блок кондиционера

Наружный блок кондиционера состоит из следующих основных узлов:

Вентилятор, создающий поток воздуха для обдува конденсатора.
Конденсатор — это радиатор, в котором происходит охлаждение и конденсация фреона, воздух проходящий мимо конденсатора нагревается и уходит в окружающую среду. Компрессор, осуществляющий сжатие хладагента и поддержание его движения по холодильному контуру. Плата управления устанавливается, как правило, в инверторных кондиционерах. В неинверторных моделях всю электронику стараются размещать во внутреннем блоке.

Внутренний блок кондиционера

Внутренний блок состоит из следующих основных узлов:

Переносчиком тепловой энергии в кондиционере (как и в холодильнике) является специальный хладагент. Чаще всего им является фреон.

Испаряясь, хладагент (как впрочем и любая другая жидкость) отбирает тепло. Вы можете провести такой физический опыт: протрите руку спиртом или спиртосодержащим раствором (например одеколоном). Вы почувствуете холод. Это спирт испаряясь отбирает тепло вашего тела.

Следует отметить, что использовать кондиционер в качестве обогревателя чрезвычайно выгодно. При потреблении 1 кВт электроэнергии кондиционер перенесет в квартиру около 3 кВт тепла. Помимо этого, воздух в помещении не будет пересушиваться.

принцип работы кондиционера

Основной принцип работы мульти сплит системы

Мульти сплит системы отличаются наличием двух, сообщающихся между собой замкнутых блоков, внешнего и внутреннего. Принцип работы мульти сплит системы очень схож с предыдущим, отличием является то, что внешний блок – компрессорно-конденсаторный, а внешний – испарительный. Главным отличием таких систем является наличие функции обогрева, т.е. весь процесс может обращаться в обратном направлении, что позволяет фреону не только забирать температуру, но и отдавать ее.

Источники

Источник — http://www.xiron.ru/content/view/2167/28/
Источник — http://kondicionershik.ru/printsip-raboty-konditsionera-v-kvartire/
Источник — http://www.mitsubishi.ru/mitsubishi-electric/stati/princip-raboty-i-ystroystvo-condicionera/
Источник — http://js. com.ua/articles/princip_rabot_rondicionera/
Источник — http://www.asamagroup.ru/articles/14518.html
Источник — http://www.projectclimat.ru/info/articles/chto_takoe_konditsioner_printsip_rabotyi_konditsionera/
Источник — http://oventilyatsii.ru/princip-raboty-kondicionerov.html
Источник — http://www.mosng.ru/articles/chto-takoe-konditsioner/
Источник — http://stroyteplo.by/index.pl?act=NEWSSHOW&id=2015042803
Источник — http://tehnika-soveti.ru/kak-rabotaet-split-sistema/

Особенности гидравлических режимов систем отопления | C.O.K. archive | 2011

2011-06-15

14754 1 0

Опубликовано в журнале СОК №6 | 2011

Rubric:

  • Heating

Тэги:

  • Heating radiators

В статье рассмотрены некоторые гидравлические особенности работы систем отопления, показаны способы их выявления и учета в практике проектирования. Обусловлена необходимость детальной визуализации технических параметров гидравлического расчета. Дано описание новых систем и стояков повышенной протяженности с тупиковым и попутным движением теплоносителя.

Сложившая практика проектирования систем отопления рекомендует тупиковые системы отопления применять ограниченно, при этом протяженность тупика не должна превышать пять-шесть стояков или этажей. При гидравлическом расчете неувязка гидравлических сопротивлений в одном тупике допускается до 15 %. Внедрение автоматического гидравлического расчета позволяет обеспечить гидравлическую увязку циркуляционных колец через каждый нагревательный прибор, применяя установку регулирующих клапанов или дросселирующих устройств. При данном варианте гидравлическая регулировка систем отопления может быть обеспечена достаточно легко. Однако, как показывают расчеты, для гидравлической увязки в ряде случаев необходимо устанавливать дросселирующие устройства с отверстиями диаметром менее 3–5 мм, а при таких размерах отверстия возникает серьезная опасность его засорения или зарастания, отложившимися солями. Следует заметить, что при подборе терморегулирующих вентилей для участков, на которых требуемый размер дросселя менее 3–5 мм, применение терморегулирующих вентилей также недопустимо. Щель между клапаном и седлом будет 0,2–0,4 мм, что значительно меньше, а следовательно, вероятность засорения повышается. Это обстоятельство накладывает свои требования к проектированию систем отопления: ❏ увеличение диаметров подающих и обратных трубопроводов; ❏ последовательное размещение нескольких дросселирующих устройств; ❏ разделение подающего или обратного трубопроводов по длине (высоте), а может быть одного из них, на секции, в пределах которых диметры дросселирующих устройств и характеристики терморегулирующих вентилей гарантируют их незасоряемость; ❏ обязательное переоборудование тупиковых стояков в стояки с попутным движением теплоносителя; ❏ совместное использование выше указанных мероприятий, что позволяет наиболее эффективно манипулировать преимуществами каждого мероприятия. Увеличение диаметра подающих и обратных трубопроводов ведет к уменьшению скорости теплоносителя, а это, в свою очередь, оказывает существенное влияние на режим воздухоотделения, образования воздушных пробок, нарушающих тепловую устойчивость, и на конструктивные изменения системы. Известно, что в зависимости от скорости теплоносителя пузырьки газа могут всплывать, находиться во взвешенном состоянии и, наконец, увлекаться потоком воды. Критическими скоростями потоков воды для вертикальных труб являются скорости 0,2–0,25 м/с, а в наклонных и горизонтальных трубах — 0,1–0,15 м/с. В горизонтальных и наклонных трубах пузырьки газа занимают верхнее положение. С увеличением скорости движения воды до 0,6 м/с начинается дробление газовых скоплений, пузырьки в верхней части труб, отрываясь от их поверхности, двигаются по криволинейным траекториям. При скорости движения воды более 1 м/с мелкие пузырьки постепенно распространяются по всему сечению труб — и в трубопроводе возникает т.н. «газоводяная эмульсия».Отделение пузырьков от теплоносителя необходимо осуществлять при скорости потока менее 0,1 м/с. Скорости теплоносителя в вертикальных трубопроводах порядка 0,2–0,25 м/с, а в горизонтальных — 0,1–0,15 м/с, являются критическими, здесь возможно образование неподвижных воздушных пробок, что нарушает гидравлический режим. Все это указывает на то, что при решении проблем обезвоздушивания систем отопления необходимо иметь скоростные данные теплоносителя на каждом участке — избегать критических скоростей в трубопроводах и дополнительно согласовывать их уклоны с условиями опорожнения трубопроводов. Уклоны следует делать, если это необходимо, при малых скоростях, как по направлению движения теплоносителя, так и против, а при повышенных скоростях — только по ходу теплоносителя. Критически малые скорости теплоносителя, как правило, следует ожидать в верхних коллекторах радиаторов, особенно если размер коллектора радиатора больше диаметра подводящего патрубка. В этом случае возможно скапливание воздуха в верхней части коллектора — из-за различия диаметров коллектора и подводки, и исключение из активного теплообмена части поверхности теплообмена радиатора. Такой радиатор следует устанавливать под некоторым углом, обеспечивающем размещение верхней кромки полости коллектора, по крайней мере, на одном уровне с верхом воздухоотводного отверстия. Установка последовательно нескольких дросселирующих устройств дает незначительный эффект. Так, при последовательной установке двух дросселирующих устройств их диаметры увеличиваются только в 1,189 раза, при трех — в 1,316, при четырех — в 1,414, при пяти — в 1,495 раза. Из приведенных цифр наглядно видно, что возможности данного мероприятия крайне ограничены.При определении гидравлических характеристик регулирующих устройств (на подводках к нагревательным приборам для тупикового стояка) может оказаться, что требуемая область работы устройства вызывает опасение его засорения. Это наиболее вероятно для приборов нижних этажей. Возникает вопрос, как обеспечить условия безопасного регулирования тепловой производительности этих приборов. Одним из возможных вариантов как раз является разделение подающего или обратного трубопроводов стояка, а может быть одного из них, по длине (высоте) на секции, в пределах которых диаметры дросселирующих устройств и характеристики терморегулирующих вентилей гарантируют безопасность работы этих устройств (рис. 1, 2 и 3). Конструктивно это выполняется в следующем порядке. Рассчитывается полностью тупиковый стояк, определяются требуемые размеры дросселирующих устройств. Из условия допустимости минимального размера щели дросселирующего устройства определяется последний нагревательный прибор. Означенную таким образом группу нагревательных приборов отдельными трубопроводами присоединяем к разводящим магистральным трубопроводам. Проводим аналогично гидравлический расчет уже укороченного стояка, далее из условия допустимости минимального размера щели дросселирующего устройства также определяем последний нагревательный прибор укороченного стояка. Определившуюся группу нагревательных приборов отдельными трубопроводами присоединяем к магистральным разводящим трубопроводам и т.д. Таким образом, большой стояк преобразуется в несколько последовательно расположенных стояков, в каждом из которых обеспечиваются нормальные условия регулирования теплоотдачи нагревательных приборов. Для повышения компактности образовавшегося стояка один из трубопроводов стояка (подающий или обратный) может быть выполнен единым, как это показано на рис. 1, 2 и 3. Другим, на наш взгляд, оригинальным способом повышения эффективности и надежности регулируемости тупиковых стояков является использование принципа организации попутного движения теплоносителя, конструктивное выполнение чего приведено, например, на рис. 4. Подобные схемы разнятся только видом трубопровода, используемого для организации попутности движения. Независимо от протяженности стояка для всех нагревательных приборов обеспечиваются примерно одинаковые гидравлические условия работы. Правда, некоторым недостатком, как этого технического решения, так и предыдущего, является наличие дополнительного трубопровода. Однако, техническое решение, связанное с использованием в стояках попутного движения, дает неоспоримые преимущества системам отопления с нижней разводкой подающей и обратной магистралей, и позволяет отказаться от применяемой в настоящее время системы с верхней разводкой подающего трубопровода и с прокладкой обратного трубопровода по подвалу. В этом случае нет необходимости прокладывать разводящий магистральный трубопровод под потолком обитаемого помещения. Третий трубопровод стояка может быть использован для дополнительного обогрева строительных конструкций. Рассматривая комбинационные варианты выполнения стояков, следует обратить внимание на стояки, использующие сочетание тупиковой части стояка со стояком выполненного с попутным движением теплоносителя. Один из возможных вариантов компоновки таких стояков приведен на рис. 5. Попутность движения теплоносителя в части стояка осуществлена за счет изменения трассировки трубопровода обратного стояка. Возможно аналогичное выполнение за счет трассировки трубопровода подающего стояка. Таким образом, в статье рассмотрены некоторые гидравлические особенности работы систем отопления, показаны способы их выявления и учета в практике проектирования. В статье также показана необходимость детальной визуализации технических параметров гидравлического расчета. Дано описание новых систем отопления с нижней разводкой подающей и обратной разводящих магистралей и стояков повышенной протяженности с тупиковым и попутным движением теплоносителя.

Дроссельное устройство — обзор

ScienceDirect

РегистрацияВход

Масло из стационарного дроссельного устройства поступает в камеру регулирования давления и камеру давления.

Из: Теория электрогидравлического управления и ее приложения в экстремальных условиях, 2019 г. Дроссельная эффективность

Эффективность дросселирующих устройств не определяется напрямую, поскольку они по своей природе являются диссипативными. Тем не менее, иногда они определяются для удобства или для возможности сравнения компонентов.

Энергоэффективность дроссельной заслонки можно определить как отношение полной выходной энергии к полной входной энергии:

(2.46)ηen=m˙houtm˙hin

отношение полезного выхода эксергии к общему входу эксергии:

(2.47)ηex=m˙exoutm˙exin=1−Exdm˙exin

Для операции адиабатического дросселирования, которая часто имеет место, эффективность использования энергии часто составляет 100%. Эксергетическая эффективность ниже 100% из-за необратимости, связанной с неограниченным расширением.

Просмотреть главуКнига покупок

Прочитать главу полностью

URL: https://www.sciencedirect.com/science/article/pii/B9780124172036000028

Howard L. Freese, in Fermentation and Biochemical Engineering Handbook (Third L. Freese, in Fermentation and Biochemical Engineering) , 2014

11.0 Поиск и устранение неисправностей систем испарения

Иногда возникает необходимость изучить работу испарителя, чтобы оценить его работу в других условиях эксплуатации или определить, почему система работает не так, как ожидалось. К счастью, большинство условий, которые приводят к тому, что испаритель не соответствует ожидаемой производительности, легко исправить. Поэтому поиск и устранение неисправностей часто означает проверку мелких деталей, которые сильно влияют на производительность системы испарителя. Конечно, возможно, что тип испарителя был использован неправильно, предоставленная поверхность теплопередачи не соответствует предполагаемому назначению, или произошло загрязнение.

Несоответствия в работе могут быть вызваны отклонениями физических свойств жидкостей, скорости потока, температур на входе, механической конструкцией оборудования или проблемами, возникшими во время установки оборудования. Специалист по устранению неполадок должен сначала проверить, соответствуют ли составы, потоки, температуры и физические свойства тем, которые указаны для проекта. Затем он должен изучить чертежи оборудования, чтобы определить, может ли проблема заключаться в способе изготовления оборудования или в способе его установки. После того, как эти основные элементы будут рассмотрены, в приведенном ниже контрольном списке перечислены некоторые вопросы, которые следует задать.

Каландрии:

1.

Вентилируется ли паровая сторона для удаления воздуха или других захваченных газов?

2.

Правильно ли подобран паровой регулирующий клапан? Каково фактическое давление пара в паровой камере?

3.

Правильно ли выбран конденсатоотводчик и правильно ли он подобран?

4.

Регулирующий клапан и конденсатоотводчик работают правильно?

5.

Паровой конденсат заливает часть поверхности? Какова температура парового конденсата? Сопло для конденсата достаточно большое? Трубопровод конденсатоотводчика имеет достаточный размер?

6.

Поддерживается ли уровень технологической жидкости в надлежащем месте? Откалиброваны ли приборы для измерения уровня жидкости? Провода прибора забиты?

7.

Достаточна ли задержка жидкости для предотвращения помпажа?

8.

Соответствуют ли технологические составы и температуры использованным при проектировании? Содержит ли технологический материал достаточное количество летучих веществ для обеспечения адекватного кипения?

9.

Какова температура верхней части каландров с естественной циркуляцией? Температура выше температуры жидкости может указывать на неадекватную циркуляцию по какой-то причине?

10.

Соответствует ли доступное давление пара расчетному?

11.

Подходят ли технологические сопла?

12.

Достаточно ли продувается или продувается технологическая сторона?

13.

Были ли удалены мусор и другие посторонние предметы с оборудования и трубопроводов перед запуском? Как часто чистится агрегат? Как выглядит оборудование до чистки? Адекватная ли уборка?

14.

Если предоставляется насос, соответствуют ли насос и система? Насос кавитирует?

15.

Обеспечено ли достаточное противодавление для предотвращения закипания технологической среды, когда этого требует работа испарителя (тип с погружной трубой)?

16.

Происходит ли унос? Сепараторы уноса правильно подобраны и установлены? Они подключены?

17.

Важно ли разбавление?

18.

Достаточны ли расходы для поддержания режимов потока, используемых в проекте? Падение давления не соответствует норме?

19.

Для устройств с падающей пленкой, является ли устройство отвесным? Равномерно ли распределяется жидкость по каждой трубке? Вентилируется ли впускной канал для удаления вскипавших паров? Достаточны ли потоки для образования пленки? Достаточен ли расход на выходе для предотвращения разрыва пленки?

Конденсаторы:

1.

Предусмотрена ли система вентиляции постоянного давления? Правильно ли он установлен? Инертные вещества следует вводить после конденсатора, а не перед ним?

2.

Адекватна ли система вентиляции?

3.

Правильно ли подобраны патрубки для конденсата? Попадает ли жидкость в конденсатор? Если они горизонтальны, трубы расположены ровно (или наклонены к выпускному отверстию)?

4.

Все ли трубопроводы подходят?

5.

Водяная сторона работает под вакуумом?

6.

Соответствуют ли температуры и состав используемым при расчете?

7.

Достаточно ли расход воды? Правильно проветривается?

8.

Был ли удален мусор или другие посторонние предметы с оборудования и трубопроводов перед запуском?

9.

При воздушном охлаждении подходит ли впускной трубопровод для обеспечения хорошего распределения? Правильно ли установлены лопасти вентилятора? Выдают ли двигатели номинальную мощность? Ремни вентилятора проскальзывают? Заметна ли рециркуляция горячего отработанного воздуха?

Проверка производительности — это экспериментальная процедура, помогающая понять производительность испарительной системы. Тесты могут быть выполнены для выявления и характеристики неудовлетворительной производительности и часто указывают методы улучшения работы. Также могут потребоваться эксплуатационные испытания, чтобы установить, что новая система испарителя соответствует характеристикам, гарантированным поставщиком. Испытания можно использовать для определения производительности испарителя в различных условиях эксплуатации или для получения данных для проектирования новой испарительной системы. Американский институт инженеров-химиков опубликовал процедуру под названием Процедура испытаний испарителей [21]. Эта процедура охватывает методы проведения тестов производительности и обсуждает несколько факторов, влияющих на производительность и точность результатов тестирования.

Испытания проводятся для определения производительности, скорости теплопередачи, экономии пара, потерь продукта и циклов очистки. Практически все критерии производительности испарителя получены из различий тестовых измерений. Ошибки могут возникать при измерении расхода, температуры и давления, концентрации и качества пара. Факторы, которые могут иметь большое влияние на производительность, включают разбавление, вентиляционные потери, тепловые потери и физические свойства жидкостей.

К частым причинам низкой производительности испарительной системы относятся следующие:

Низкая экономия пара:

Экономия пара при фиксированной схеме подачи можно рассчитать на основе теплового и материального балансов. Экономия пара ниже рассчитанной при проектировании агрегата может быть результатом одного или нескольких из следующих факторов:

1.

Утечка уплотняющей воды сальника насоса

2.

Excessive rinsing

3.

Excessive venting

4.

Flooded barometric condensers

5.

Dilution from condensate leakage

Low Rates of Heat Transfer:

1.

Поверхности, покрытые солью, накипью или загрязнение0003

4.

Inadequate circulation

Excessive Entrainment:

1.

Air leakage

2.

Excessive flashing

3.

Sudden изменения давления

4.

Несоответствующие уровни жидкости

5.

Несоответствующие уровни давления

6.

Работа с повышенной производительностью

Короткое время между циклами очистки:

Время простоя, необходимое для очистки, может не соответствовать ожидаемой частоте очистки. Короткие циклы могут быть вызваны:

1.

Внезапными изменениями рабочих условий (таких как давление или уровень жидкости)

2.

Низкий уровень жидкости

5
4.
4.
Введение воды или других загрязняющих веществ во время очистки, промывки или из-за негерметичности уплотнения

4.

Высокая температура различия

5.

Неправильная процедура очистки

. article/pii/B9781455725533000118

Чао Тан, Фэн Донг, в справочном модуле по биомедицинским наукам, 2021 г. Вентури, сопла. В расходомере дифференциального давления используется дроссельное устройство, установленное в прямой трубе для создания изменения площади поперечного сечения, как правило, в форме сжатия-расширения. Жидкость в трубе будет сжиматься при прохождении через дросселирующее устройство в соответствии с уравнением Бернулли:

(11)P1+12ρu12+mgh2=P2+12ρu22+mgh3

где P – статическое давление, u – скорость жидкости, ρ – плотность жидкости, г ускорение свободного падения, ч – высота, м – масса жидкости, нижние индексы 1 и 2 представляют параметры жидкости при полном поперечном сечении трубы и суженной трубе. Для сплошной и несжимаемой жидкости одинаковой высоты (горизонтальная труба) м и ρ являются постоянными, а ч 1 = ч 2 . Таким образом, скорость потока увеличится, а давление уменьшится в месте установки дросселирующего устройства из-за суженной площади поперечного сечения. Следовательно, перепад давления Δ P , вызванный сжатием из-за дросселирующего устройства, равен:

(12)ΔP=P1−P2=12ρu22−u12

Введем объемный расход Q  =  u 1 A 1 = u 2 A 2 в уравнение (12), мы можем иметь общую корреляцию измерения расхода с перепадом давления Δ P .

(13)W=CdA11−β42ΔPρ

где C d — коэффициент расхода для корректировки корреляции для различных дросселирующих устройств и приложений, β = A2/A1 — отношение эквивалентного диаметра. Типичное дросселирующее устройство включает в себя трубку Вентури, диафрагму, V-образный конус и т. д., как показано на рис. 2. Существуют также другие варианты дросселирующих устройств для различных применений (Tan et al., 2013).

Рис. 2. Типичные дросселирующие устройства: (A) трубка Вентури, (B) диафрагма и (C) V-образный конус.

Расходомеры перепада давления широко применяются в однофазном потоке из-за их простой конструкции и стандартных моделей. Однако они столкнулись с некоторыми проблемами при измерении многофазного потока, поскольку процесс течения намного сложнее, чем однофазный поток в процессе сжатия. Для решения таких задач было предложено много новых моделей, таких как модели газожидкостного потока, модели потока влажного газа, модели потока жидкость-жидкость (Lin, 1982; Тан и Донг, 2010).

Простой способ приравнять многофазный поток к однофазному состоит в том, чтобы заменить плотность плотностью смеси ρ m в уравнении. (13), отнесенное к однородной модели:

(14)W=CdA1−β42ΔPρL1+ρLρG−1xG

где x G – массовая доля газа в газожидкостном двухфазном потоке. Однако эта форма не работает должным образом, потому что несколько фаз не идеально смешаны, а перепад давления, создаваемый несколькими фазами, не всегда соответствует идеальной модели (уравнение). (14). В газожидкостном двухфазном потоке газ и жидкость текут раздельно, поэтому расход газа и жидкости равен:

(15)WG=CdA1−β42ΔPGρG=CdAG1−β42ΔPρG

(16)WL=CdA1−β42ΔPLρL=CdAL1−β42ΔPρL

where Δ P G and Δ P L — перепад давления газа и жидкости, протекающих по трубе по отдельности при том же массовом расходе, что и в двухфазном потоке (Чисхолм, 1967). A G и A L площади потока газа и жидкости соответственно с А Г  +  А Л  =  А . Переставьте уравнения. (15), (16) можно установить следующее соотношение:

(17)ΔPΔPG=ΔPLΔPG+1

Таким образом, модель раздельного течения:

(18)WG=CdA1−β42ΔPρGx+1−xρ2/ ρ1

Эту модель также можно использовать для измерения влажного газа с высокой объемной долей газа. Существует также множество других моделей аналогичной формы для оценки расхода газа и жидкости (Gupta et al., 2016; Lin, 19).82).

Для двухфазного потока жидкость-жидкость, такого как двухфазный поток нефть-вода, нормальная модель дифференциального давления аналогична газожидкостному потоку, например, модели однородного потока в уравнении. (14). Но следует провести специальную обработку, чтобы учесть влияние вязкости жидкости, которая напрямую влияет на падение давления потока жидкости. Затем пересмотренное уравнение (13) можно выразить как (Tan and Dong, 2010):

(19)Wm=CdA2ΔPρw1−x1−β41+ρwρo0,5μoμw0,2/χ+1/χ2

, где χ — коэффициент Локхарта-Мартинелли (Л-М), характеризующий перепады давления двухфазного потока, определяемый как: потеря давления δP , связанная с потерей давления на трение и потерей напора. Благодаря сложному анализу и моделированию этот δP также может прогнозировать общий расход и компенсировать измерение Δ P . Типичным примером является конический расходомер с длинной талией, показанный на рис. 3: 9.0003

Рис. 3. Конусный расходомер с длинной талией.

В этой схеме массовая доля масла x O можно определить по формуле:

(21)xO=ρOρW12−nY+ρOρW12−n

(22)Y=aρOρW12-n-12-FbF-1

(23)F=Cd2ρWΔP1-β4Cpd22-n2ρWδP12-n

где C pd

9043 коэффициент расхода для прогнозирования расхода δP и параметр a и b могут быть приспособлены для условий двухфазного потока нефть-вода для получения более точных прогнозов.

(24) WM = CDA1 — β4ε2Δpρww1 — WOA/χ+1

Просмотреть книгу Глава покупки

Читать полная глава

URL: https://www.sciendirect.com/science/article/pii/b9780128254866746746746746746746746746746746746746746746746746746746746746746746746746746746746746746746464646464646464646464646464646н646464646н646464646н.

Теория электрогидравлического управления и ее приложения в экстремальных условиях, 2019

2 Выводы

1.

Электрогидравлический сервоклапан с компенсационным дросселем может обеспечить разумное соотношение диаметра отверстия и длины компенсирующего дросселя за счет оптимизации конструкции параметров дросселирующего устройства и достижения лучших динамических характеристик.

2.

Согласно модели передаточной функции электрогидравлического сервоклапана с компенсирующим дросселем, модель электрогидравлического сервоклапана может быть проанализирована с помощью регулируемого параметра сопротивления жидкости. Результаты моделирования показывают, что размер камеры, образованной новой конструкцией, мало влияет на работу клапана.

3.

Теория, метод и результаты этой статьи могут быть использованы в качестве теоретической основы для проектирования и анализа нового электрогидравлического сервоклапана.

Просмотр книги Глава покупки

Читать полная глава

URL: https://www.sciendirect.com/science/article/pii/b9780128140567000136

Alison Subiantoro, KIM TIA, KIM TIA, KIM TIA, KIM TIA, KIM TIA, KIM TIA, KIM TIA.

Расширитель с роликовым поршнем

В последние несколько лет в связи с потребностью в более высокой энергоэффективности холодильных систем были предложены расширители для восстановления работы расширения хладагентов, которая обычно теряется в дроссельных устройствах. Сообщалось о примерно 50% и 10% улучшении коэффициентов полезного действия (COP) транскритических CO 2 и обычных (включая R22, R134a и т. д.) систем охлаждения, соответственно (Subiantoro & Ooi, 2013). Экспандер можно рассматривать как компрессор, работающий в обратном цикле. Он нагнетает жидкость под высоким давлением в машину и расширяет ее для производства энергии. Были испытаны различные типы механизмов для применения в расширителях, включая конструкцию с катящимся поршнем.

Большинство моделей, сформулированных в этой главе, по-прежнему применимы, за исключением того, что термодинамический процесс стал обратным. Всасывающая камера подвергается всасыванию при высоком давлении с последующим расширением, а нагнетательная камера выпускает хладагент при низком давлении. Процесс расширения снижает температуру, в отличие от компрессора. Эти изменения требуют корректировки конструкции системы смазки.

Одной из неотъемлемых проблем экспандера с вращающимся поршнем является механизм управления всасывающим потоком, хотя для нагнетательного порта клапан не требуется. Пластинчатый клапан нельзя использовать в расширителе, потому что закрытие и открытие всасывающего клапана определяют изменение давления в расширителях, а не наоборот, как в компрессоре. Можно использовать электромагнитный клапан, но время срабатывания должно быть достаточно быстрым. Кроме того, электромагнитный клапан стоит дорого. Другое решение состоит в том, чтобы установить две машины с роликовым поршнем последовательно (Zhao, Li, Ma, Liu, & Zhang, 2014). Первый расширитель меньше и используется исключительно для процесса всасывания, а второй используется для расширения хладагента до более низкого давления и для выпуска хладагента. При таком расположении клапан не нужен. Недостаток, очевидно, в том, что нужно две машины. Эти проблемы делают механизм с вращающимся поршнем менее популярным для расширительных приложений по сравнению, например, с вращающимися лопастями или спиральными механизмами.

Просмотреть главуКнига покупок

Прочитать главу полностью

URL: https://www.sciencedirect.com/science/article/pii/B9780128169988000108

T.J. Катона, в книге «Понимание и смягчение последствий старения атомных электростанций», 2010 г.

Модель ВВЭР-440/230

Общий дизайн и концепция ВВЭР-440/230 были основаны на предположении о наихудшем случае разрыва трубы. подключен к основному контуру. Эти трубы были оснащены дросселирующими устройствами для ограничения размера разрыва поперечным сечением с эквивалентным диаметром 32 мм. кроме того, системы безопасности конфайнмента были спроектированы таким образом, чтобы справиться с этой небольшой аварией с потерей теплоносителя (LOCA). Система локализации ВВЭР-440/230 имела небольшую способность к избыточному давлению, а ее характеристики герметичности были неудовлетворительными и не обладали необходимой способностью ограничивать выбросы в случае аварии с потерей среднего или большого размера. У станций с ВВЭР-440/230 были и другие проблемы и проблемы безопасности, например. внутренние и внешние опасности не были должным образом учтены при проектировании, избыточность, разнообразие и разделение систем безопасности не были оптимизированы, а уязвимость к отказам по общей причине (CCF) и т. д. была неприемлемой. Вопросы безопасности АЭС с ВВЭР-440/230 подробно обсуждаются в отчете Международного агентства по атомной энергии (МАГАТЭ) (IAEA-TECDOC-640, 1992). Следует отметить, что, как и все реакторы с ВВЭР-440, они обладают определенными неотъемлемыми характеристиками безопасности, которые превосходят большинство современных станций PWR, например. прочная конструкция основных узлов и трубопроводов, в том числе главных циркуляционных линий системы охлаждения реактора, изготовленных из аустенитной нержавеющей стали. на всех АЭС с ВВЭР в разной степени были решены проблемы эксплуатации и безопасности путем переоснащения и внесения изменений в конструкцию. При оценке общей безопасности АЭС с ВВЭР-440/230 можно сделать вывод, что первоначальный проект станции имел неадекватные системы для преодоления аварий, которые постулируются в качестве проектной основы западных PWR. хотя реакторы ВВЭР-440/230 доказали присущие им запасы безопасности, а операторы внедрили программы модернизации, остается сомнительным, достигнут ли такие станции уровня безопасности, который существует в реакторах PWR западного дизайна той же марки.

Основным недостатком ВВЭР-440/230 с точки зрения длительной эксплуатации в соответствии с проектом была высокая облученность стенки корпуса реактора (КР) быстрыми нейтронами и относительно быстрое охрупчивание материала КР. . Проблема усугублялась отсутствием надлежащей программы наблюдения за вирусом ЧП на этих заводах. Было предпринято несколько попыток оценить охрупчивание материала основания и сварного шва этих корпусов корпусов, но, несмотря на это, достоверность оценки старения этих корпусов остается под вопросом. хотя были реализованы масштабные и успешные программы повышения безопасности, блоки 1–4 АЭС «Козлодуй», Болгария, блоки 1 и 2 АЭС «Бохунице», Словакия, станции с ВВЭР-440/230 были остановлены. В отличие от этого, Кольские 1 и 2 и Нововоронешские 3 и 4 блоки в России уже получили лицензии на эксплуатацию еще на пять лет после реализации программ модернизации и повышения безопасности (см., например, Росэнергоатом, 2003). Следовательно, долгосрочная эксплуатация и управление сроком службы станции типа ВВЭР-440/230 не может рассматриваться как общая практика и будет обсуждаться ниже лишь в ограниченной степени. Однако опыт эксплуатации, накопленный на АЭС с ВВЭР-440/230, широко использовался при изменении связанных со старением проектных и эксплуатационных соображений новых АЭС с ВВЭР. Поэтому будут обсуждаться важные аспекты старения станций с ВВЭР-440/230.

View chapterPurchase book

Read full chapter

URL: https://www.sciencedirect.com/science/article/pii/B9781845695118500196

Robert T. Balmer, in Modern Engineering Thermodynamics, 2011

6.6 Дроссельные устройства

Дроссельное устройство — это общее название любого устройства или процесса, который просто рассеивает энергию давления m˙pv путем необратимого преобразования ее в тепловую энергию. В отличие от сопел и диффузоров дросселирующие устройства не обеспечивают рекуперации полезной энергии. Они просто преобразуют энергию давления в тепловую энергию посредством процессов диссипативного вязкого течения (обычно турбулентного). Фактически, любое устройство, которое подвергается большому необратимому падению давления, можно рассматривать как дросселирующее устройство. На рис. 6.5 схематично показаны различные распространенные дросселирующие устройства.

Рисунок 6.5. Некоторые распространенные дросселирующие устройства.

Дроссельным устройством можно считать любое аэргономическое устройство, основной целью которого является обеспечение сопротивления потоку. Дроссели могут быть или не быть изолированы. Но обычно они представляют собой такие маленькие устройства и имеют такие высокие скорости потока, что время пребывания жидкости в них слишком мало для того, чтобы происходил значительный перенос тепла и энергии. Следовательно, дросселирующее устройство обычно считается адиабатическим независимо от того, изолировано оно на самом деле или нет.

Небольшой физический размер большинства дросселирующих устройств также препятствует значительному изменению удельной потенциальной энергии между входным и выходным потоками. Однако дроссель не обязательно должен иметь одинаковые скорости потока на входе и выходе, и, следовательно, он может иметь значительное изменение удельной кинетической энергии на нем.

Следовательно, мы определяем дросселирующее устройство со следующим набором термодинамических условий:0003

Применяя эти условия к модифицированному балансу скорости энергии по уравнению. (6.12) дает

0−0+m˙[hin−hout+(Vin2−Vout2)/2gc+0]=0

или

(6.23)hout=hin+(Vin2−Vout2)/2gc

Если Vin =Vout, так как когда жидкость несжимаема, а площади входа и выхода дросселя равны (например, случаи a–d на рис. 6.5), то уравнение (6.23) сводится к более простому виду

(6.24)hout=hin

Такие дросселирующие устройства называются изоэнтальпическими (т. е. имеют постоянную энтальпию).

Даже если входная и выходная скорости явно не равны в какой-то задаче, вы все равно сможете обосновать это с помощью более простого уравнения. (6.23) в результате вашего анализа. Высокоскоростной поток неравноплощадного дросселирующего устройства всегда ограничен скоростью звука в протекающей среде. 5

Следовательно, если ч велико, скажем, порядка 1000 БТЕ/фунт (2300 кДж/кг), то удельная кинетическая энергия потока никогда не может быть больше, чем 2 или 3% от этой значения и поэтому может считаться незначительным. Эмпирическое правило, обсуждавшееся ранее в этой главе, можно применить следующим образом: 9.0048 Если вам дали задачу о дроссельном устройстве без адекватной информации о скорости и где скорость не является неизвестной, которую вы должны найти как часть решения , , тогда вы должны предположить, что условия удельной кинетической энергии либо равны (и поэтому аннулируют друг друга) или что они пренебрежимо малы .

Для несжимаемой жидкости, протекающей через дроссельное устройство, мы можем использовать уравнение. (6.19) в уравнении (6.23) для получения

c(Tin-Tout)+v(pin-pout)+(Vin2-Vout2)/2gc=0

, и если мы пренебрежем членами удельной кинетической энергии (или будем иметь Vin=Vout), то это уравнение можно изменить, чтобы получить

Tout=Tin+(v/c)(pin-pout)

и начиная с p in обычно больше, чем p out , это уравнение говорит нам, что обычно происходит повышение температуры в несжимаемой жидкости, протекающей с незначительным изменением удельной кинетической энергии через дросселирующее устройство.

Для идеального газа с постоянной удельной теплоемкостью мы можем заменить уравнение. (6.22) в уравнение (6.23) чтобы получить

Tout=Tin+(Vin2−Vout2)/(2gccp)

Это уравнение говорит нам, что в случае незначительного изменения удельной кинетической энергии дросселирование идеального газа является изотермическим процессом.

Фактическая температура на выходе из дросселирующего устройства для чистого вещества зависит от его коэффициента Джоуля-Томсона μ Дж , определяемого как

(6,25) мкДж=(∂T/∂p)ч μ J определяется полностью с точки зрения интенсивных термодинамических свойств, это также является интенсивным термодинамическим свойством. Процесс дросселирования, имеющий незначительное изменение удельной кинетической энергии, называется процессом постоянного ч , поэтому коэффициент Джоуля-Томсона для любого чистого вещества можно аппроксимировать по данным, полученным при таком дросселировании, как p out p in , тогда Δ p обычно является отрицательным числом для такого процесса. Ясно, что положительное значение для μ J означает, что при таком дросселировании температура падает (Δ T = T out T in < 0) и отрицательное значение для μ J означает, что температура увеличивается. Для изотермического процесса дросселирования (такого, как в идеальном газе) μ Дж = 0,

Чистое газообразное вещество, имеющее положительный коэффициент Джоуля-Томсона, может подвергаться непрерывному понижению температуры и в конечном итоге сжижается правильно разработанным процессом дросселирования. Это было основой процесса, введенного в 189 г.5 Карла фон Линде (1842–1934) для крупномасштабного производства жидкого воздуха. Температура, при которой мкм J = 0 для настоящего чистого вещества называется его Температура инверсии T Inv и мкм J > 000488 J > 00048 8888 88 88 88 88 88 88 88 88 88 88 88 88 88 88 88 88 88 88 88 88 88 88 88 88 88 88 88 88 88 88 88 88 88 88 88 88 88 88 88 88 88 88 88 88 88 88 88 88 88 88 88 88 88 88 88 8 88 8 881 Дж. инв. и μ J < 0 для T > T инв. . Таким образом, температура реального газа в процессе дросселирования снижается, если его температура на входе меньше температуры его инверсии. Однако температуру газа нельзя понизить за счет эффекта Джоуля-Томсона, если температура газа на входе превышает его «максимальную» температуру инверсии (см. табл. 6.3). 6

Таблица 6.3. The Maximum Joule-Thomson Inversion Temperature for Various Common Gases

Substance Maximum Inversion Temperature
K R
Air 659 1186
Argon 780 1404
Углекислый газ 1500 2700
Гелий 40 72
Hydrogen 202 364
Neon 231 416
Nitrogen 621 1118
Oxygen 764 1375

Источник : перепечатано с разрешения издателя Zemansky, MW, Abbott, MM, Van Ness, HC, 1975. Basic Engineering Thermodynamics, второе изд. Макгроу-Хилл, Нью-Йорк.

На рис. 6.6 показано изменение коэффициента Джоуля-Томсона в зависимости от давления и температуры для воздуха и двуокиси углерода.

Рисунок 6.6. Изменение коэффициента Джоуля-Томсона воздуха и углекислого газа в зависимости от давления и температуры.

Просмотреть главуКнига покупок

Прочитать главу полностью

URL: https://www.sciencedirect.com/science/article/pii/B9780123749963000063

Richard Circuit MarkellEditor, in Analog13 Design, 200003

Введение

Эффективность в области преобразования энергии подобна прозрачности в области передачи света. Неудивительно поэтому, что усилители класса D часто называют прозрачными, так как они не имеют значительных потерь мощности. В отличие от усилителей класса D, коммутирующих почти без потерь, усилители классов от A до C являются дросселирующими устройствами, которые расходуют значительное количество энергии. Усилители «низших классов» (А-С) моделируются как реостаты (переменные резисторы), тогда как усилители класса D моделируются как вариаки (переменные трансформаторы). Идеальный резистор рассеивает мощность; тогда как идеальный трансформатор — нет. Подобно трансформаторам (вариакам), многие усилители класса D могут передавать энергию в обоих направлениях — вход на выход и выход на вход.

Усилители класса D также могут игнорировать реактивную нагрузку, что может быть сверхъестественным. Усилитель класса D, работающий с выходом переменного тока, потребляет очень небольшую дополнительную входную мощность, когда на его выходе размещается значительная емкостная или индуктивная нагрузка. Это связано с тем, что на реактивной нагрузке есть переменное напряжение и переменный ток, протекающий через нее, но угол сдвига фаз напряжения и тока таков, что реальная мощность не рассеивается. Усилитель класса D в конечном итоге переключает мощность между своим входом и выходом, делая и то, и другое с минимальными потерями. Идеальный усилитель класса D можно рассматривать как не имеющий места для рассеивания мощности, поскольку все его компоненты не имеют потерь; то есть он не содержит резисторов.

Просмотреть главуКнига покупок

Прочитать главу полностью

URL: https://www.sciencedirect.com/science/article/pii/B9780123978882000389

8.1 Характеристики и принцип работы одноступенчатого перепускного клапана со сбалансированным поршнем и фиксированным отверстием

8.1.1 Конструктивные характеристики

Как показано на рис. 8.1, одноступенчатый перепускной клапан со сбалансированным поршнем и фиксированным отверстием состоит из клапана сердечник, седло клапана, пружина и другие компоненты. Сердечник клапана имеет специальную новую структуру в форме тыквы, а сердечник клапана и седло клапана образуют камеру регулирования давления, камеру измерения давления и фиксированное отверстие.

Рисунок 8.1. Одноступенчатый перепускной клапан со сбалансированным поршнем и фиксированным отверстием, используемый в самолетах.

1.

Небольшая шаровая головка сердечника клапана и отверстие в седле клапана образуют динамическое демпфирующее отверстие, образуя фиксированное дроссельное устройство; седло клапана имеет геометрическую полость, и могут быть реализованы ощущение динамического давления и обратная связь.

2.

Сердечник клапана имеет сбалансированный поршень и ударопрочную заднюю часть, и в то же время играет направляющую роль в рабочем состоянии. Верхняя поверхность сердечника конического клапана с шаровой головкой имеет сферическую форму. Сердечник клапана и седло пружины расположены за счет сферического контактного позиционирования, а сердечник клапана и осевая передача усилия пружины находят центр сами по себе.

3.

Конструкция золотника с изогнутым профилем улучшает угол и импульс вытекающего потока жидкости и компенсирует гидравлическую силу.

8.1.2 Принцип работы

Принципиальная схема работы одноступенчатого перепускного клапана со сбалансированным поршнем и фиксированным проходным сечением показана на рис. 8.2. Одноступенчатый перепускной клапан со сбалансированным поршнем и фиксированным отверстием используется для создания давления в системе и контроля стабильности, поддерживая рабочее давление в системе.

Рисунок 8.2. Принципиальная схема одноступенчатого перепускного клапана со сбалансированным поршнем и фиксированным отверстием.

1.

Когда гидравлический насос начинает работать, сердечник клапана обычно находится в закрытом состоянии. Он перекрывает проход между напорным маслом и обратным трубопроводом, чтобы остановить поток напорного масла гидравлического насоса и установить давление в системе за счет мнимого заполнения камеры впуска масла.

2.

Когда масляный контур системы заполнен маслом, давление масла постепенно повышается. Масло из неподвижного дросселирующего устройства поступает в камеру регулирования давления и напорную камеру. Давление, воспринимаемое областью поршня A камеры измерения давления, сравнивается с давлением предварительно установленной силы пружины, образуя простую петлю обратной связи по нагрузке, тем самым толкая клапан и сжимая пружину. Когда скорость потока масла слишком велика, давление масла повышается. Сердечник клапана открывается и соединяет контур высокого давления с контуром возврата. Масло под давлением сбрасывается в масляный бак, чтобы ограничить повышение давления масла в системе, чтобы реализовать стабильный контроль давления и обеспечить нормальное рабочее давление в системе. Чем больше жесткость пружины, тем большее изменение давления соответствует определенной величине открытия. Чем меньше жесткость пружины, тем меньшее давление соответствует открытию; чем плавнее давление, тем лучше характеристики давления-расхода.

3.

Функция фиксированного отверстия состоит в том, чтобы формировать масляный бак, формировать динамическую гидравлическую пружину, ограничивать поток масла под давлением в датчике давления, замедлять скорость сердечника клапана (закрытого или открыт), увеличивают динамическое демпфирование и обеспечивают стабильность работы и динамические характеристики клапана.

4.

Одноступенчатый перепускной клапан со сбалансированным поршнем и фиксированным отверстием служит в качестве предохранительного клапана для ограничения максимального давления масла в определенном масляном контуре или системе. Клапан обычно представляет собой закрытый клапан регулирования давления (также известный как предохранительный клапан). Сердечник клапана закрывается пружиной, и масло под давлением из определенного масляного канала или системы воздействует на сердечник клапана. Когда давление слишком высокое, сердечник клапана открывается, и масло под давлением выливается в масляный бак.

8.1.3 Рабочие характеристики
1.

Снижение вибрации. Колебания потока от гидравлического насоса или нагрузки вызывают внезапное повышение или понижение управляющего давления после дросселирования фиксированным отверстием, которое ощущается камерой измерения давления. После замедления колебаний давления давление сравнивается с предварительно установленным усилием пружины. Управляющее давление косвенно управляет открытием клапана, тем самым выполняя функцию самодемпфирования и буферизации. Для перепускного клапана прямого действия сердечник клапана непосредственно воспринимает управляющее давление, а управляющее давление напрямую сравнивается с усилием пружины, поэтому функция подавления вибрации отсутствует.

2.

Шумоизоляция. Течение жидкости осуществляется по поверхности геометрического пространства, состоящего из сердечника микроклапана и корпуса клапана. После входа в клапан жидкость образует сплошной слой на поверхности. Структура поверхности и уравновешивающий поршень могут частично устранить шум потока, создаваемый дросселированием.

3.

Стабилизация напряжения. Неподвижное дроссельное устройство и камера измерения давления образуют динамическую гидравлическую пружину, которая значительно ослабляет колебания давления, вызванные колебаниями потока, и динамически контролирует стабильность давления.

View chapterPurchase book

Read full chapter

URL: https://www.sciencedirect.com/science/article/pii/B9780128140567000082

In Handbook of Liquefied Natural Gas, 2014

4.2 .5 Идеальные ожижители Линде-Хэмпсона

На рис. 4-8 показан идеальный ожижитель Линде-Хэмпсона, первоначально изобретенный Карлом фон Линде и Уильямом Хэмпсоном независимо друг от друга в 1895 году для сжижения воздуха, и соответствующая диаграмма температура-энтропия (T-s). В идеальной системе сжижения сжижаемый газ сжимается в процессе многоступенчатого адиабатического сжатия и многоступенчатого изобарического охлаждения. Поэтому этот процесс рассматривается как изотермическое сжатие при температуре окружающей среды (1-2). Газ высокого давления охлаждается в теплообменнике (2-3) и затем изоэнтальпически расширяется в дросселирующем устройстве (3-4). Жидкая (6) и газообразная фазы (5) разделяются в сепараторе фаз, а несжиженный газ используется для охлаждения теплого потока высокого давления в теплообменнике. Температура обратного газа низкого давления на выходе из теплообменника зависит от эффективности используемого теплообменника. В случае идеального теплообменника с эффективностью теплообменника 100 % T 7 = Т 1 .

РИСУНОК 4-8. Идеальный процесс сжижения Линде-Хэмпсона.

Для количественного определения доли газа, который сжижается (выход жидкости ( y ) в идеальном процессе Линде-Хэмпсона), y кг газа сжижается из 1 кг исходного газа после сжатия, охлаждения и расширение. Принимая контрольный объем, включающий теплообменник, расширительный клапан и фазовый сепаратор, закон сохранения энергии дает

(4-18)1⋅h3=yh6+(1−y)h2

Therefore, the liquid yield y can be expressed as

(4-19)y=h2−h3h2−h6

where h 1 , h 2 , and h 6 представляют собой энтальпию в соответствующих точках состояния 1, 2 и 6. — Нужны ли дросселирующим устройствам клапаны?

Задавать вопрос

спросил

Изменено 1 год, 3 месяца назад

Просмотрено 65 раз

$\begingroup$

Почему дроссель всегда имеет ведущий к нему клапан? Если в замкнутом системном контуре непосредственно перед дросселирующим устройством существует охлажденный газ, то не должно ли внезапное уменьшение площади поперечного сечения уменьшать давление без клапана так же хорошо, как и с ним?

Возможно, мое непонимание заключается в работе клапана. Является ли он в целях безопасности и оставлен открытым до тех пор, пока не потребуется аварийное закрытие, или он действительно открывается и закрывается в соответствии с таймером или датчиком в схеме, чтобы поддерживать его «синхронизацию» с положением жидкости, циркулирующей через этапы. цикл системы (например, испарение, конденсация и т. д.)?

РЕДАКТИРОВАТЬ: В частности, мой вопрос связан с холодильным циклом. Назначение клапана — изолировать жидкость на ведущей к ней ступени на некоторое время, а затем выпускать ее на следующую. Например, клапан между конденсатором и испарителем. Насколько я понимаю, он запускается, когда компрессор выполняет свою работу до тех пор, пока все возможное тепло не рассеется в конденсаторе, и ТОЛЬКО ТОГДА клапан открывается, чтобы другая половина контура (которая была как бы выкачана компрессором) … то есть от клапана до компрессора из-за его низкого давления создается ПАДЕНИЕ давления (компрессор->контур клапана, ведущий к контуру клапан->компрессор), правильно? И что после того, как клапан открывается, компрессор выключается до тех пор, пока все возможное тепло не будет взято из испарителя и он не будет готов к сжатию, когда, конечно же, клапан снова закроется и компрессор запустится. Промывать, повторять до бесконечности!

  • термодинамика
  • энергия
  • гидродинамика
  • конденсированное состояние
  • давление

$\endgroup$

2

$\begingroup$

Я предполагаю, что под «клапаном» вы имеете в виду расширительный клапан. Это комбинация простого дросселирующего устройства («крохотное отверстие в препятствии в контуре») с механизмом управления, позволяющим регулировать мощность охлаждения («диаметр отверстия») в зависимости от нагрузки.

Ваше заблуждение, похоже, связано с тем, что холодильник/кондиционер на самом деле является непрерывно работающей машиной. Это не похоже на двигатель отто, который имеет разные, разделенные во времени технологические этапы в течение одного цикла (впрыск топлива/всасывание воздуха, зажигание, выпуск), но все «этапы» термодинамического процесса холодильника/кондиционера происходят в разных местах цепь, все в то же самое время. Дроссельное устройство снижает давление в направлении потока в одном месте контура, в то время как компрессор снова увеличивает давление в другом месте. Конденсатор снижает температуру среды (отдавая тепло) и увеличивает ее плотность в направлении потока еще в одном месте, в то время как испаритель повышает температуру (забирая тепло из окружающей среды) и снижает плотность снова в другом месте.

Таким образом, управляемый клапан (кроме самого дросселирующего устройства) определенно не нужен для поддержания процесса, в отличие от клапанов в оттодвигателе, которые вообще гарантируют, что эта штука работает.

$\endgroup$

12

$\begingroup$

Картинка поможет. Кроме того, вы, кажется, предполагаете, что дроссельное устройство может дать вам необходимый перепад давления при различных условиях эксплуатации. Это, вероятно, не так . .. отсюда и клапан вверх по течению. Этот входной клапан позволяет выдерживать различное падение давления и делает связанную систему «настраиваемой».

$\endgroup$

3

Твой ответ

Зарегистрируйтесь или войдите в систему

Зарегистрируйтесь с помощью Google

Зарегистрироваться через Facebook

Зарегистрируйтесь, используя электронную почту и пароль

Опубликовать как гость

Электронная почта

Требуется, но не отображается

Опубликовать как гость

Электронная почта

Требуется, но не отображается

Нажимая «Опубликовать свой ответ», вы соглашаетесь с нашими условиями обслуживания, политикой конфиденциальности и политикой использования файлов cookie

термодинамика — Почему дроссельный клапан является энтальпийным, а сопло — нет?

спросил

Изменено 1 год, 1 месяц назад

Просмотрено 561 раз

$\begingroup$ 92_{in}}{2} $$ Приведенное выше уравнение верно для обоих устройств . Теперь мой учебник (Термодинамика Ценгеля и Болеса) объясняет без особых оснований, что для дроссельных клапанов мы имеем это $V_{out}\приблизительно V_{in}$. Это означает, что $ h_{in}\ приблизительно h_{out}$ подразумевает, что дроссельные клапаны нетривиально снижают давление, но не изменяют заметно скорость.

Моя проблема в том, что автор утверждает, что для форсунок аппроксимация $V_{in}\приблизительно V_{out}$ не выполняется. Таким образом, форсунки не изоэнтальпийны, а дроссельные клапаны. Почему это так? Почему форсунки расходуют давление для увеличения скорости, в то время как дроссельные клапаны расходуют давление, но не увеличивают скорость? Если я представлю отдельные частицы жидкости, движущиеся в сопло, я могу представить, как они должны ускоряться, чтобы обеспечить постоянную массовую скорость потока. Точно так же, если я представлю частицы жидкости, проходящие через дроссельный клапан (т. е. пористую пробку, поэтому частицы должны проходить через перфорированное препятствие), я также могу визуализировать, как они должны ускоряться, когда они проходят через перфорированную пористую пробку, чтобы гарантировать, что жидкость массовый расход остается постоянным. Итак, в конечном счете, мой вопрос заключается в следующем: почему дроссельные клапаны могут нетривиально снижать давление без заметного изменения скорости в условиях установившегося потока, в то время как сопло, вызывающее нетривиальное снижение давления, должно заметно изменять скорость?

  • термодинамика
  • гидродинамика
  • статистическая механика
  • давление
  • температура

$\endgroup$

$\begingroup$

Внутренняя конструкция форсунки сильно отличается от внутренней конструкции дроссельного клапана или пористой пробки. Сопло выполнено с гладким плавно изменяющимся поперечным сечением, так что поток почти изоэнтропический, с dh=vdP. Дроссельный клапан или пористая заглушка имеет негладкие извилистые поперечные сечения, что приводит к крайне необратимому потоку и большому вязкому рассеянию. Так, в случае сопла мы ожидаем существенных изменений температуры за счет изоэнтропического расширения и сжатия, а в дросселе или пористой пробке, по крайней мере для идеального газа, падение давления не приводит к понижению температуры.

Кроме того, массовый расход через дроссельный клапан или пористую заглушку намного ниже, чем через эквивалентное сопло, так что кинетическая энергия газа на входе и выходе намного ниже, чем у эквивалентного сопла, а изменение кинетической энергии пренебрежимо мал по сравнению со случаем сопла.

$\endgroup$

6

$\begingroup$

В качестве примера возьмем трубу диаметром 8 дюймов. Фланцы устанавливаются в месте, где должен быть установлен дроссельный клапан, дроссельный клапан устанавливается между этими фланцами, и дроссельный клапан обычно имеет тот же размер, что и труба, в которой он установлен, с соответствующей обвязкой клапана для обеспечения требуемой падение давления для данного применения. Когда жидкость проходит через клапан, скорость жидкости существенно увеличивается, когда жидкость проходит через трим клапана, но эта жидкость выходит обратно в 8-дюймовую линию после дроссельного клапана. Это означает, что если вы берете контрольный объем через весь дроссельный клапан, скорость на выходе равна скорости на входе для несжимаемого потока, а скорость на выходе немного выше, чем скорость на входе для сжимаемого потока.

Аналогичный анализ для сопла ясно показывает, что скорость на выходе из сопла существенно выше скорости на входе в то же сопло как для сжимаемого, так и для несжимаемого потоков. Тот же вывод будет сделан, если вы возьмете контрольный объем со стороны входа дроссельного клапана и распространите этот контрольный объем только на трим клапана, но этот конкретный контрольный объем выбирается редко (если вообще когда-либо). Это приводит к тому, что автор опубликовал относительно скоростей входа и выхода обоих устройств.

$\endgroup$

2

$\begingroup$

Доброе утро:

Возможно, я опоздал с ответом, но вот что:

Причина, по которой некоторые жидкости охлаждаются при расширении, внутренне обусловлена ​​взаимодействием их молекул.

Идеальное дросселирование/расширение приводит к другому уровню давления при сохранении той же энтальпии и, что более важно, той же температуры.

Однако, когда происходит реальное дросселирование/расширение, столкновения между частицами могут быть слабее, что приводит к меньшей средней кинетической энергии. Жидкость подвергается так называемому отрицательному эффекту Джоуля-Томсона (отрицательное отклонение от идеального дросселирования).

Вот почему вы можете увидеть иней возле дроссельных клапанов в циклах чиллера. Я схватил одну из них рукой, и мне действительно стало холодно!

$\endgroup$

Твой ответ

Зарегистрируйтесь или войдите в систему

Зарегистрируйтесь с помощью Google

Зарегистрироваться через Facebook

Зарегистрируйтесь, используя электронную почту и пароль

Опубликовать как гость

Электронная почта

Требуется, но никогда не отображается

Опубликовать как гость

Электронная почта

Требуется, но не отображается

Нажимая «Опубликовать свой ответ», вы соглашаетесь с нашими условиями обслуживания, политикой конфиденциальности и политикой использования файлов cookie

.

Дроссельное устройство | Измерение расхода по перепаду давления

Дроссельное устройство также называется дроссельным расходомером. Типичный расходомер дифференциального давления. Обычно используется для измерения расхода газа, жидкости и пара в промышленном производстве.

Дроссельное устройство представляет собой проточное устройство, в котором жидкость, заполненная трубой, протекает по трубе. Поток будет образовывать локальное сужение при дросселировании. Это увеличит скорость потока и снизит статическое давление. Таким образом, перепад давления создается до и после дросселирования (или называется дроссельным расходомером).

Sino-Inst, производитель расходомеров дифференциального давления. Включая диафрагму, трубки Вентури, Annubar и т. д. Подходит для измерения расхода жидкости, газа и пара. Пожалуйста, свяжитесь с нашими инженерами по продажам для получения технической поддержки!

Особенности дросселирующего устройства

  • Дроссельное устройство имеет преимущества простой конструкции, прочности, надежной работы, стабильной работы, высокого класса и низкой цены. Следовательно, объем дросселирующего устройства имеет преимущество по сравнению с другими расходомерами.
  • Широкий диапазон диаметров труб, φ2~φ3000 мм (или больше). Допускается форма поперечного сечения, круглая или прямоугольная.
  • Стандартное дросселирующее устройство может определять свою степень измерения без калибровки реального расхода.
  • Применяется для широкого спектра тестируемых сред. Его можно использовать практически для всех измерений расхода газа, пара и жидкости.
  • Рабочее давление может достигать 32 МПа. Может также использоваться для отрицательного давления.
  • Диапазон температур среды: -185~+650℃. Другие расходомеры пока невозможны.
  • Конструкция нестандартного дросселирующего устройства разнообразна. Почти применимо к различному измерению расхода жидкости.
  • Диапазон расхода можно изменить на месте, установив диапазон преобразователя дифференциального давления.
  • Он прост в использовании, эксплуатации, простоте в освоении, а ежедневное обслуживание невелико.

Расширенное чтение: Принцип работы расходомера V-Cone

Тип и состав дросселирующего устройства

Дроссельное устройство состоит из дросселирующих частей (таких как диафрагма, сопло и т. д.). Датчики давления, крепежные устройства (фланцы, болты, гайки, прокладки и т. д.) и другие аксессуары (трубки для направления давления, запорный клапан и т. д.). Он также может быть оснащен передними и задними измерительными трубками и фланцами на обоих концах прямого участка трубы.

Throttle Parts

9 79 S9
Code  Name Code Name Code Name
A Restrictor orifice K Wide Edge Orifice T Machined Вентури
B Стандартное или восьмищелевое отверстие L Сопло Вентури U Клиновой расходомер
C Long diameter nozzle M Small nozzle V V cone flowmeter
D End orifice plate N Orifice plate W Rough welded iron plate venturi
E 1/4 дуговое отверстие (сопло) O Эксцентрическое отверстие x Шоровидный оттенок x Шоростный впадение x Шоровидный оттенок. 0879 P ISA1932 nozzle or eight-slot nozzle Y Double orifice plate
G High pressure lens aperture plate Q Round hole plate Z Averaging tube flowmeter
H Кольцевое отверстие R Коническое входное отверстие
J Ветромер крыльчатого типа Double venturi

Pressure method

Code Name Code Name
H Corner connection pressure (ring chamber) J Diameter pressure
Z Угловое соединение, давление (сверление) T Особое давление
F Фланец, давление

Note: The current limiting orifice has no such representation method

Diameter

Code 2 3 4 5 6 8 10 12 15 17 20 22 25
Diameter (mm) 25 32 40 50 65 80 100 125 150 175 200 225 250
Code 27 30 32 35 37 40 45 50 60 70 80 90 99
Diameter (mm) 275 300 325 350 375 400 450 500 600 700 800 900 1000

Pipe diameter greater than 1000 is expressed as 1/10 of the actual pipe diameter

Nominal pressure

Code 0 1 2 3 4 6 10 16 20 25 32 40
Nominal pressure(Mpa) 0. 6 1 1.6 2.5 4 6.4 10 16 20 25 32 42

Специальное номинальное давление заполняется в соответствии с фактическим значением расчетного давления

Способ подачи

9 Комплект поставки99999
Код
A Прибор в сборе с передней 10D и задней 5D измерительными трубками (включая соединительный фланец и технологический фланец)
A1 Прибор в сборе с передней 10D и задней 5D измерительной трубкой (включая соединительный фланец)
A2 Устройство в сборе с передней 10D и задней 5D измерительной трубкой (конец трубы с перевернутым скосом)
B Полный комплект оборудования с измерительными трубками на входе и выходе (включая технологические фланцы) в соответствии со стандартами нефтехимии (HGJ516- 87)
B1 В соответствии с нефтехимическим стандартом (HGJ516-87) полный комплект оборудования с входными и выходными измерительными трубами (конец трубы с обратным скосом)
B2 Согласно нефтехимическому стандарту (HGJ551 87), полный комплект оборудования без передних и нижних измерительных трубок
С По стандарту Минэнерго (ГД87-1101) полный комплект оборудования с входными и выходными измерительными трубками
D Фланцы, кольцевые камеры или фланцы напорные, дроссели, напорные трубы, крепежные детали и другие комплекты приборов без мерных трубок
E Дроссель одинарный
1 9:09421 9:09421 9:09421 9:09421 Принцип работы и цена] SI3051ANB Расходомер Annubar

Принцип измерения расхода дросселирующим устройством

Дроссельное устройство искусственно вызывает дросселирование в трубопроводе, по которому циркулирует среда (как показано на рисунке). Когда измеряемая среда протекает через дросселирующее устройство, она вызывает локальное сжатие. Поток концентрируется, скорость потока увеличивается, а статическое давление уменьшается.

Таким образом, возникает перепад давления между входной и выходной сторонами дроссельной заслонки. Существует определенная функциональная связь между этим перепадом давления и расходом. Чем больше поток, тем больше создаваемый перепад давления. Следовательно, расход можно измерить путем измерения перепада давления.

Вам может понравиться: Как работает преобразователь давления?

Типы дросселирующих устройств

Как наиболее важная часть расходомера, дросселирующие устройства делятся на две категории: стандартные дросселирующие устройства и нестандартные дросселирующие устройства.

Стандартные дросселирующие устройства включают: стандартную диафрагму, стандартное сопло ISA, сопло с длинной шейкой, классическую трубку Вентури и сопло Вентури;

К нестандартным дросселирующим устройствам относятся: кольцевая диафрагма. Форсунка четверть круга. Четверть круга диафрагмы. Коническая входная диафрагма. Отсутствует круглая диафрагма. Эксцентриковая диафрагма. Пластина с двойным отверстием. Расходомерная трубка с низкими потерями давления. Прямоугольная трубка Вентури, V-конусный расходомер, клиновой расходомер, встроенная диафрагма, ограничительная диафрагма и т. д.

Расширенное чтение​: Что такое сопло потока?

Расходомер с дроссельной диафрагмой

Расходомер с диафрагмой работает вместе с диафрагмой, датчиком перепада давления, по принципу измерения перепада давления. Простая установка. Расходомеры с диафрагмой могут использоваться с газами, жидкостями, агрессивными и высокотемпературными жидкостями. Расходомеры с диафрагмой могут измерять пар, газ, жидкости и многие промышленные области. Области применения включают расход пара, питательную воду для котлов и расход жидкости в водопроводных сетях зданий.

Расширенное чтение: Встроенный расходомер DP|Газ, жидкость, пар|Компактная конструкция

Вам может понравиться:

Компания Sino-Inst предлагает дросселирующие устройства. Вам доступны различные дроссельные устройства, в том числе латунь, углеродистая сталь. Вы также можете выбрать один из бесплатных образцов.

Sino-Inst является поставщиком дросселирующих устройств. Расходомеры с диафрагмой наиболее популярны на внутреннем рынке, в Юго-Восточной Азии и на Среднем Востоке.

Вы можете обеспечить безопасность продукции, выбирая сертифицированных поставщиков, в том числе 375 с ISO9001, ISO14001 и другие сертификаты.

Запросить предложение

KimGuo11DP Расходомеры, расходомеры

Шесть дросселирующих устройств для холодильного оборудования. — чиллеры |Водяные чиллеры, воздухоохладители,винтовые/спиральные/центробежные чиллеры,промышленные | Чиллеры HVAC

Перейти к содержимому

Предыдущий Следующий

  • Посмотреть увеличенное изображение

Функция дросселирующего механизма: дросселирование и снижение давления жидкого хладагента высокого давления для обеспечения разницы давлений между конденсатором и испарителем, чтобы хладагент в испарителе испарялся при требуемом низком давлении для достижения цели охлаждение; Расход хладагента, подаваемого в испаритель, адаптируется к изменению тепловой нагрузки испарителя.

Чиллер с воздушным охлаждением GESON

Типы дросселирующих устройств можно разделить на:

(1) Ручной расширительный клапан;

(2) Расширительный клапан с плавающим шаром;

(3) Терморегулирующий клапан;

(4) Электронный расширительный клапан;

(5) Капилляр;

(6) Дроссельная короткая трубка;

 

1. Ручной расширительный клапан:

Используется только для аммиачной холодильной системы, экспериментального устройства, резервного байпаса и т. д.

 

Воздухоохладитель GESON спирального типа

 

2. Расширительный клапан с плавающим шаром:

Помимо дросселирования, снижения давления и регулировки расхода, он также может поддерживать определенный уровень жидкости в испарителе. Применимо к: испарителям со свободной поверхностью жидкости, таким как:

 Затопленный испаритель;

 Резервуар для хранения циркулирующей жидкости низкого давления;

 Интеркулер.

 

 

Особенности: простая структура; большие колебания уровня жидкости в поплавковой камере и большая ударная сила, передаваемая поплавком на сердечник клапана, который легко повредить.

 

3. Терморегулирующий клапан:

Принцип работы: Открытие клапана регулируется перегревом газообразного хладагента на выходе из испарителя. Используется для: незаполненного испарителя. Тип: внутренний баланс, внешний баланс.

 

Расширительный клапан

 

Давление, создаваемое внешне уравновешенным терморасширительным клапаном, является давлением на выходе испарителя; давление, создаваемое внутренне сбалансированным терморасширительным клапаном, является выходным давлением расширительного клапана.

 

Внешний уравновешивающий терморегулирующий клапан в основном используется в системах, где потери давления испарения или перепад давления велики, гидравлическое сопротивление велико, испарительный змеевик длинный, колебания температуры велики, а давление испарения после дросселирования значительно превышает давление на выходе из испарителя.

 

Для иллюстрации рассмотрим случай: две идентичные системы охлаждения, расширительный клапан 1 отбалансирован изнутри, расширительный клапан 2 отбалансирован снаружи, а перегрев установлен на 8k. После запуска системы расширительный клапан начинает регулироваться автоматически. Если выход испарителя действительно перегрет до 8К, внешний уравновешивающий ТРВ будет продолжать работать в этом состоянии. Из-за падения давления в испарителе внутренний уравновешивающий клапан ощущает давление. Если он слишком высок, клапан необходимо закрыть. Когда внутренний балансировочный клапан чувствует, что он 8к, то реальный перегрев на выходе из испарителя уже больше 8к, что приведет к неполной загрузке испарителя.

 

При выборе и подборе терморегулирующих клапанов учитывайте:

 Тип хладагента;

 Диапазон температур испарения;

 Максимальная холодопроизводительность испарителя после клапана;

 Разность давлений до и после клапана;

 

4.

Электронный расширительный клапан

Электронный расширительный клапан использует электрический сигнал, генерируемый регулируемым параметром, для управления напряжением или током, подаваемым на расширительный клапан, для достижения цели регулировки подачи жидкости. Диапазон регулировки подачи жидкости широк, а реакция регулировки быстрая.

 

Управляющий сигнал электронного расширительного клапана

(1) Регулировка перегрева: используется для сухих испарителей для установки датчика температуры или датчика давления на выходе из испарителя для сбора перегрева хладагента на выходе из испарителя и обратной связи для регулировки открытие регулирующего клапана;

Составная регулировка с прямой и обратной связью позволяет устранить запаздывание регулирования перегрева, вызванное теплоемкостью стенки испарительной трубы и датчика, улучшить качество регулирования системы и контролировать перегрев в заданном диапазоне в широком диапазоне температур испарения. ;

(2) Назначенные процедуры настройки: размораживание блока теплового насоса, контроль температуры нагнетания компрессора.

(3) Регулировка уровня жидкости: используется для затопленного испарителя.

 

5. Капилляр

Тонкая красная медная трубка диаметром 0,7–2,5 мм и длиной 0,6–6 м широко используется в небольших полностью закрытых устройствах прямого охлаждения. Емкость подачи жидкости зависит от: состояния хладагента на входе в капилляр (давление, температура) и геометрии капилляра (длина, внутренний диаметр).

 

 

 

Характеристики капилляра:

 Простая структура и низкая цена;

 Без движущихся частей;

 Систему не нужно оборудовать резервуаром для жидкости, а заправка хладагентом невелика;

 После остановки компрессора давление может быстро достичь равновесия, что снижает пусковую нагрузку двигателя;

 Плохое качество регулировки, подача жидкости не регулируется при изменении условий работы;

 Подходит для случаев, когда температура испарения не сильно меняется, а рабочие условия относительно стабильны.

 

6. Дроссельная короткая трубка

Дроссельная короткая трубка представляет собой дроссельное устройство с дроссельным отверстием фиксированного сечения, которое используется в некоторых автомобильных кондиционерах, чиллерах и тепловых насосах.

 

 

Основные преимущества: низкая цена, простота изготовления, хорошая надежность, простота установки, отсутствие датчика температуры, используемого для оценки увеличения охлаждающей нагрузки в системе терморегулирующего клапана и т. д., с хорошей взаимозаменяемость и способность к самобалансировке.

Перейти к началу

Процесс дросселирования — Изентальпийский процесс | Определение

Процесс дросселирования  – это термодинамический процесс  , в котором энтальпия газа или среды остается постоянной (h = const) . Процесс дросселирования является одним из изоэнтальпийных процессов . В процессе дросселирования никакая работа не выполняется системой или над ней (dW = 0), и обычно нет теплопередачи ( адиабатическая ) из системы или в систему (dQ = 0).

С другой стороны, процесс дросселирования не может быть изоэнтропическим. Это принципиально необратимый процесс . Характеристики процесса дросселирования:

  1. Без переноса работы
  2. Без переноса тепла
  3. Необратимый процесс
  4. Изентальпический процесс

Дросселирование потока вызывает значительное снижение давления из-за дросселирования устройства 0047 местная потеря давления . Дросселирования можно добиться, просто введя сужение в линию, по которой течет газ или жидкость. Это ограничение обычно делается с помощью частично открытого клапана или пористой пробки. Такие потери давления обычно называют незначительными потерями , хотя они часто составляют большую часть потерь тепла. Незначительные потери примерно пропорциональны квадрату расхода, , и поэтому их можно легко интегрировать в уравнение Дарси-Вейсбаха через Коэффициент сопротивления К .

Например, рассмотрим дросселирование идеального газа , протекающего через частично открытый клапан. Из опыта можно заметить, что: p в > p вых , v в < v вых , где p давление и v

0 — удельный объем 5 . Мы также можем заметить, что удельные энтальпии остаются прежними, т. е. h в = h из .

Удельная энтальпия равна удельной внутренней энергии системы плюс произведение давления на удельный объем.

h = u + pv

Следовательно, если давление уменьшается, удельный объем должен увеличиваться, если энтальпия остается постоянной (при условии, что u постоянна). Поскольку массовый расход постоянен, изменение удельного объема наблюдается как увеличение скорости газа , что также подтверждается наблюдениями.

Если есть изменение внутренней энергии u, то должно произойти изменение температуры . Обычно температура жидкости падает. Однако в особых случаях температура может оставаться прежней или повышаться. Для снижения давления в системе можно использовать частично открытый клапан или пористую пробку.

Эффект Джоуля-Томсона – коэффициент Джоуля-Томсона

Изменения температуры в процессе дросселирования подвержены эффекту Джоуля-Томсона . При комнатной температуре и нормальном давлении все газы, кроме водорода и гелия , охлаждают при расширении газа. Охлаждение происходит потому, что необходимо совершить работу, чтобы преодолеть дальнодействующее притяжение между молекулами газа, когда они удаляются друг от друга. Эффект зависит от значения коэффициента Джоуля-Томсона , который определяется как:0050 используется для снижения давления и снижения температуры хладагента от давления на выходе из конденсатора до более низкого давления, существующего в испарителе.

Дроссель влажного пара

Влажный пар характеризуется качеством пара в диапазоне от нуля до единицы – открытый интервал (0,1). Дросселирование влажного пара также связано с сохранением энтальпии . Энтальпия сохраняется, потому что система не совершает никакой работы (dW = 0) и обычно отсутствует передача тепла (адиабатическая) из системы или в нее (dQ = 0). Но в данном случае снижение давления вызывает повышение качества пара . Когда давление падает, часть жидкости во влажном паре испаряется, что повышает качество пара (т. е. долю сухости). Этот процесс происходит потому, что температура насыщения ниже при более низком давлении. Более низкая температура, более низкое давление, более качественный пар содержит ту же энтальпию, что и исходный пар.

Пример: дросселирование влажного пара

Ступень паровой турбины высокого давления работает в установившемся режиме при входных условиях  6 МПа, t = 275,6°C, x = 1 (точка C). Пар выходит из этой ступени турбины при давлении 1,15 МПа, температуре 186 °С и x = 0,87 (точка D). Определить паросодержание пара при дросселировании от 1,15 МПа до 1,0 МПа. Предположим, что процесс адиабатический и система не совершает никакой работы.

See also: Steam Tables

Solution:

The enthalpy for the state D must be calculated using vapor quality:

h D, wet = h D, пар x + (1 – x ) h D, жидкость  = 2782 . 0,87 + (1 – 0,87) . 790 = 2420 + 103 = 2523 кДж/кг

Поскольку это изоэнтальпический процесс, мы знаем энтальпию для точки T. Из паровых таблиц мы должны найти качество пара, используя то же уравнение и решив уравнение для качества пара, x:

ч T, wet = h T,vapor x + (1 – x ) h T,liquid

x = ( h T, wet – h T, liquid ) / ( h T, vapor – h Т, жидкость ) = (2523 – 762) / (2777 – 762) = 0,874 = 87,4 %

до 87,4%, а температура снижается со 186°С до 179°С. 0,9°C. Дросселирование влажного пара обычно вызывает повышение качества пара, энтропии и снижение температуры.

 

 

Ссылки:

Ядерная и реакторная физика:

  1. Дж. Р. Ламарш, Введение в теорию ядерных реакторов, 2-е изд., Addison-Wesley, MA (193, Reading).
  2. Дж. Р. Ламарш, А. Дж. Баратта, Введение в ядерную технику, 3-е изд., Prentice-Hall, 2001, ISBN: 0-201-82498-1.
  3. В. М. Стейси, Физика ядерных реакторов, John Wiley & Sons, 2001, ISBN: 0-471-39127-1.
  4. Гласстоун, Сезонске. Разработка ядерных реакторов: разработка реакторных систем, Springer; 4-е издание, 1994 г., ISBN: 978-0412985317
  5. WSC. Уильямс. Ядерная физика и физика элементарных частиц. Кларендон Пресс; 1 издание, 1991 г., ISBN: 978-0198520467
  6. Кеннет С. Крейн. Введение в ядерную физику, 3-е издание, Wiley, 1987, ISBN: 978-0471805533
  7. Г. Р. Кипин. Физика ядерной кинетики.